Anda di halaman 1dari 149
OG A R P 2009 FRM Practice Exams aha assem as roressios 2009 FRM Practice Exams Table of Contents 2009 FRM Level | Practice Exam - Candidate Answer Sheet 2009 FRM Level | Practice Exam ~ Questions 2009 FRM Level | Practice Exam — Answer Key 2009 FRM Level | Practice Exam — Answers & Explanations ..... 2009 FRM Full Exam FRM Practice Exam | ~ Candidate Answer Sheet 1.0.0.0 47 2009 FRM Full Exam FRM Practice Exam |— Questions 2009 FRM Full Exam FRM Practice Exam |- Answer Key 2009 FRM Full Exam FRM Practice Exam | - Answers & Explanations 2009 FRM Full Exam FRM Practice Exam Il ~ Candidate Answer Sheet .. 2009 FRM Full Exam FRM Practice Exam lI — Questions .. 2009 FRM Full Exam FRM Practice Exam ll — Answer Key 2009 FRM Full Exam FRM Practice Exam Il — Answers & Explanations wo 17 119 Introduction ‘The FRM exam is a practice-oriented examination. Its questions are derived from a combination of theory, as set forth in the core readings, and “real-world” work experience. Candidates are expected to understand risk management concepts and approaches and how they would apply to a risk manager's day-to-day activities. ‘The FRM examination is also a comprehensive examination, testing a risk professional on a number of risk management concepts and approaches. It is very rare that a risk manager will be faced with an issue that can immediately be slotted into one category. In the real world, a risk manager must be able to identify any number of risk-related issues and be able to deal wit them effectively, The 2009 FRM Practice Exams have been developed to aid candidates in their preparation for the FRM Examination in November 2009. These practice exams are based on a sample of questions from the 2007 FRM Examination and are representative of the questions that will be in the 2009 FRM Examination. Wherever necessary and possible, questions, answers and references have been updated to better reflect the topics and core readings listed in the 2009 FRM Examination Study Guide. The 2009 FRM Level | Practice Exam and the FRM Full Exam Practice Exams | and II contain 40 and 50 multiple-choice questions, respectively. Note that the 2009 FRM Level | and Full Examination will consist of a morning and afternoon session containing SO and 70 multiple- choice questions, respectively. The practice exams were designed to be shorter to allow candidates to calibrate their preparedness without being overwhelming. Copyright © 2009 Glot All rights reserve Association of Risk Professionals 1 OGARP mOrEsiouas 2009 FRM Practice Exams The 2009 FRM Practice Exams do not necessarily cover all topics to be tested in the 2009 FRM Examination. For a complete list of topics and core readings, candidates should refer to the 2009 FRM Examination Study Guide. Core readings were selected by the FRM Committee to assist candidates in their review of the subjects covered by the exam. Questions for the FRM examination are derived from the “core” readings. It is strongly suggested that candidates review these readings in depth prior to sitting for the exam. Suggested Use of Practice Exams To maximize the effectiveness of the practice exams, candidates are encouraged to follow these recommendations: ‘+ Plan a date and time to take each practice exam. Set dates appropriately to give sufficient study/review time between each practice exam and prior to the actual exam. ‘© Simulate the test environment as closely as possible. © Take each practice exam in a quiet place. © Have only the practice exam, candidate answer sheet, calculator, and writing instruments (pencils, erasers) available, © Minimize possible distractions from other people, cell phones and study material. ‘© Allocate 90 minutes for each practice exam and set an alarm to alert you when 90 minutes have passed. Complete each exam but note the questions answered after the 90 minute mark. © Follow the FRM calculator policy. You may only use a Texas Instruments BA Il Plus (including the BA II Plus Professional) calculator or a Hewlett Packard 12C (including the HP 12C Platinum) calculator. * After completing each practice exam, © Calculate your score by comparing your answer sheet with the practice exam answer key. Only include questions completed in the first 90 minutes. ©. Use the practice exam Answers & Explanations to better understand correct and incorrect answers and to identify topics that require additional review. Consult referenced core readings to prepare for exam © Pass/fail status for the actual exam is based on the distribution of scores from all candidates, so use your scores only to gauge your own progress and preparedness. ‘Copyright © 2009 Global Association of Rsk Professionals 2 All rights reserved. 2 £ S x a 2 Poy oc © a = i ir a 3 Ss a 2009 FRM Level | Practice Exam Candidate Answer Sheet QQOQOOQOHOHOHOHOHOOOOOOOO QQO0OHOOHOOOOO0OOO000000 00000009000000000008 00000000000000000000 QHQOOOQOHOH909O9O00O0000O9 QOQOOQOOOOHOOOO0O0O0000 ©0000000000060000008 00000000000000000000 3 Copyright © 2009 Global Association of Risk Professionals All rights reserved. ©OGARP 2009 FRM Practice Exams This page is intentionally left blank. Copyright © 2009 Global Association of Risk Professionals 4 All rights reserved, OGARP aM ss0cunon oF mst ProressoNALS 2009 FRM Practice Exams 2009 FRM Level | Practice Exam Questions 1, To hedge against future, unanticipated, and significant increases in borrowing rates, which of the following alternatives offers the greatest flexibility for the borrower? a. Fixed for floating swap b. Interest rate collar ©. Interest rate floor 4. Call swaption 2. An investment bank uses the Exponentially Weighted Moving Average (EWMA) technique with lambda of 0.9 to model the daily volatility of a security, The current estimate of the daily volatility is 11.5%. The closing price of the security is USD 20 yesterday and USD 18 today. Using continuously compounded returns, what is the updated estimate of the volatility? 5.44% 3.62% 2.96% 131% ance 3. Consider two stocks A and B. Assume their annual returns are jointly normally distributed, the marginal distribution of each stock has mean 2% and standard deviation 10%, and the correlation is 0.9, What is the expected annual return of stock A if the annual return of stock B Is 3%? a 29% b. 2% © 11% d. 4.7% Copyright © 2009 Global Association of Risk Professionals. 5 All ights reserved. ©OGARP 4 lam seen ask orssoAss 2009 FRM Practice Exams In pricing a derivative using the Monte Carlo method, we need to simulate a reasonable number of paths for the price of the underlying asset. Suppose we use a simple model for the return of the underlying asset: vit) = drift™2,+ vol * Vd, * et), and elt) is distributed ~ N(0,1), where drift and vol are known parameters and &; is the step size. ‘The generation of each path requires a number of steps. Which of the following describes the correct procedure? a, Generate a random number from a normal distribution N(0,1), use the inverse normal function to get e{t), which will be fed into the model to get y(t). Repeat the same procedure until you get the full desired path. b. Generate @ random number from a normal distribution N(0,1), use the cumulative normal function to get e(t), which will be fed into the model to get yit). Repeat the same procedure until you get the full desired path © Generate a random number from a uniform distribution defined in [0,1], use the inverse ‘cumulative normal function to get e(t), which will be fed into the model to get yit). Repeat the same procedure until you get the full desired path. d. Generate a random number from a uniform distribution defined in [0,1], use the cumulative normal function to get e(t), which will be fed into the model to get y(t). Repeat the same procedure until you get the full desired path Arisk manager estimates the daily variance (h,) using a GARCH model on daily returns (n): hy= ag + ayes + Ble ‘Assume the model parameter values are «tp= 0.005, « = 0.04, 8 = 0.94. The long-run annualized volatility is approximately: a. 25.00% b, 13.54% 72.72% d. 7.98% Copyright © 2009 Global Association of Risk Professionals 6 All rights reserved. 2009 FRM Practice Exams 6. Asingle stock has a price of USD 10 and a current daily volatility of 2%. Using the delta-normal method, the VaR at the 95% confidence level of a long at-the-money call on this stock over a 1-day holding period is approximately: usp 0.23 USD 1.645 usp 0.33 usp 0.16 aooe 7. Aportfolio consists of two zero coupon bonds, each with a current value of USD 10, The first bond has a modified duration of 1 year and the second has a modified duration of 9 years. The yield curve is flat and all yields are 596. Assume all moves of the yield curve are parallel shifts. Given that the daily volatility of the yield is 1%, which of the following is the best estimate of the portfolio daily VaR. at the 95% confidence level? a, USD 2,33 b, USD 1.65, c UsD0.82 d. usd 116 8. Consider the following three methods of estimating the P&L of a bullet bond: full repricing, duration (PVO01), and duration plus convexity. Ranking the estimated P&L impact of a large negative yield shock from the lowest P&L impact to the highest P&L impact, what is the ranking of the methods to estimate the P&L impact? Duration plus convexity, duration, full repricing Full repricing, duration plus convexity, duration Duration, duration plus convexity, full repricing Duration, full repricing, duration plus convexity 9. Consider a position in a 5-year receive-fixed swap that makes annual payments on a USD 100 million notional. The floating leg has just been reset. The term structure is flat at 5%, the Macaulay duration of a 5-year par bond is 4.5 years, and the annual volatility of yield changes is 100bp. Your best estimate of the swap's VaR with 95% confidence over the next month is: Usd 5.5 million USD 7.1 million Copyright © 2009 Global Association of Risk Professionals 7 All rights reserved. OGARP Lon Ass0C1ATON OF RK POFESSIONAS 2009 FRM Practice Exams 10. If the gold lease rate is higher than the risk-free rate, what is the market structure of the forward market for gold? a. Contango b. Backwardation Inversion d. Need more information to determine 11. The price of a 3-year zero coupon government bond is 85.16. The price of a similar 4-year bond is 79.81, What is the one-year implied forward rate from year 3 to year 4? a. 5.8% b, 5.5%. c 5.8% d, 6.7% 112. A portfolio manager has a bond position worth USD 100 million. The position has a modified duration of 8 years and a convexity of 150 years. Assume that the term structure is flat. By how ‘much does the value of the position change if interest rates increase by 25 basis points? a. USD -1,953,125 b. USD -1,906,250 ©. USD -2,046,875 d. UDS-2,187,500 13, Afirm is going to buy 10,000 barrels of West Texas Crude Oil. It plans to hedge the purchase using the Brent Crude futures contract. The correlation between the spot and futures prices is 0.72. The volatility of the spot price is 0.35 per year. The volatility of the Brent Crude futures price is 0.27 per year. What is the hedge ratio for the firm? a, 0.5554 b. 0.9333 1.2099 d. 0.8198 ‘Copyright © 2009 Global Association of Risk Professionals 8 All rights reserved. OGARP gat assem o mst paresis 2009 FRM Practice Exams 14. Itis June 2™ and a fund manager with USD 10 million invested in government bonds is concerned that interest rates will be highly volatile over the next three months. The manager decides to use the September Treasury bond futures contract to hedge the value of the portfolio. The current. futures price is 95.0625. Each contract is for the delivery of USD 100,000 fate value of bonds. The duration of the manager's bond portfolio in three months will be 7.8 years. The cheapest to deliver bond in the Treasury bond futures contract is expected to have a duration of 8.4 years at maturity of the contract. At the maturity of the Treasury bond futures contract, the duration of the underlying benchmark Treasury bond is 9 years. What position should the fund manager undertake to mitigate his interest rate risk exposure? Short 94 contracts Short 98 contracts Short 105 contracts Short 113 contracts aooe 15. Abond trader has bought a position in Treasury Bonds with a 4% annual coupon rate on February 15, 2015. The DVO1 of the position is USD 80,000. The trader decides to hedge his interest rate risk with the 4.5% coupon rate Treasury Bonds maturing on May 15, 2017 which has a DVO1 of .076 per USD 100 face value. To implement this hedge, approximately what face amount of the 4.5% Treasury bonds maturing on May 15, 2017 should the trader sell? usp 80,000 USD 10,500,000 USD 80,000,000 USD 105,000,000 aoce 16. Suppose that A and B are random variables, each follows a standard normal distribution, and the covariance between A and B is 0.35. What is the variance of (3A + 2B)? a. 15.10 b. 14.47 © 9.20 d. 17.20 Copyright © 2009 Global Association of Risk Professionals 8 All ights reserved. OGARP v7. 18, 19. 20, LGB ASSOCUTION OF sk PHOrESSONALS 2009 FRM Practice Exams Consider a stock price $ that follows a geometric Brownian motion dS = pS dt + B'S dz, with B strictly positive and 1a fixed value. Which of the following statements is true? Hf the drift wis negative, the price one year from now will be below today’s price. The instantaneous rate of return on the stock follows a uniform distribution c. The stock price S follows a lognormal distribution. d. This model imposes mean reversion. ‘The joint probability distribution of random variables X and Y is given by f(x. 1,2, 3, and kis a positive constant, What is the probability that X +Y will exceed 5? w/o va 1/36 Cannot be determined ange Which of the following statements regarding Hypothesis Testing is incorrect? a. Hypothesis testing is used to make inferences about the parameters of a given population on the basis of statistics computed for a sample that is drawn from that population. b. Type Il error refers to the failure to reject the null hypothesis when itis actually false. ©. The p-value decision rule is to reject the null hypothesis if the p-value is greater than the significance level. d. Allelse being equal, the decrease in the chance of making a Type | error comes at the cost of increasing the probability of making a Type Il error. If stock returns are independently identically normally distributed and the annual volatility is 30%, then the daily VaR at the 99% confidence level of a stock market portfolio is approximately: 2.41% 3.11% 4.40% 1.89% ape» Copyright © 2009 Global Association of Risk Professionals 10, All rights reserved. OGARP SLL ASSEIUTON OF mst PoTEsSNALS 2009 FRM Practice Exams 21. The current value of the S&P S00 index is 1457, and each S&P futures contract is for delivery of USD 250 times the index. A long-only equity portfolio with market value of USD 300,100,000 has beta of 1.1. To reduce the portfolio beta to 0.75, how many S&P futures contract should you sell? 618 contracts 288 contracts 574 contracts 906 contracts ‘The following information should be used for the next two questions. On January 1, a risk manager observes that the 1-year continuously compounded interest rate is 5% and storage costs of a commodity product A is USD 0.05 per quarter (payable at each quarter end). He further observes the following forward prices for product A March 5.35 June 5.90 September 5.30 December 5.22 22. Given the following explanation of supply and demand for commodity product A how would you best describe its forward price curve from June to December? rh ri Explanation a. Backwardation Excess demand for A in early summer b. Backwardation ‘Supply is expected to decline after summer ©. Contango Excess demand for A in early summer d. Contango Supply is expected to decline after summer 23. What is the annualized rate of return earned on a cash-and-carry trade entered into in March and closed out in June? a 8.9% b. 98% 35.7% d, 39.1% Copyright © 2009 Global Association of Risk Professionals 11 All rights reserved. 2009 FRM Pra e Exams 24. An investor sells a June 2008 call of ABC Limited with a strike price of USD 45 for USD 3 and buys a June 2008 call of ABC Limited with a strike price of USD 40 for USD 5. What is the name of this strategy and the maximum profit and loss the investor could incur? Bear Spread, Maximum Loss USD 2, Maximum Profit USD 3 Bull Spread, Maximum Loss Unlimited, Maximum Profit USD 3 Bear Spread, Maximum Loss USD 2, Maximum Profit Unlimited Bull Spread, Maximum Loss USD 2, Maximum Profit USD 3 25. Which of the following problems are NOT inherent disadvantages of the historical simulation approach to estimating VaR? |. Itgives too little weight to more recent observations N. For long-only portfolios, its likely to understate VaR following a recent structural increase in volatilities IM. tt always ignores the fat tails present in the distribution of returns on many financial assets IV. Because of the delta approximation, it inadequately measures the risk of nonlinear instruments a. landtlonly b. tonly cI llland IV only d. illandv only 26. A bank holds USD 60 million worth of 10-year 6.59% coupon bonds that are trading at a clean price of 101,82, The bank is worried by the exposure due to these bonds but cannot unwind the position for fear of upsetting the client. Therefore, it purchases a total return swap (TRS) in which it receives annual Libor + 100 bps in return for the mark-to-market return on the bond. For the first year, the Libor sets at 6.25% and by the end of the year the clean price of the bonds is at 99.35, The net receipt/payment for the bank in the total return swap will be: Receive USD 2.23 million Receive USD 1.93 million Pay USD 1.93 million Pay USD 2.23 million Copyright © 2009 Global Association of Risk Professionals 12 All ights reserved. OG A R P 2009 FRM Practice Exams 27. Which of the following trade(s) contain basis risk? |. Long 1,000 lots Nov 07 ICE Brent Oil contracts and short 1,000 lots Nov 07 NYMEX WTI Crude Oil contracts Hl. Long 1,000 lots Nov 07 ICE Brent Oil contracts and long 2,000 lots Nov 07 ICE Brent Oil at- the-money put Ml. Long 1,000 lots Nov 07 Brent Oil contracts and short 1,000 lots Dec 07 ICE Brent contracts IV. Long 1,000 lots Nov 07 ICE Brent Oil contracts and short 1,000 lots Dec 07 NYMEX WTI Crude Oil contracts a 1Bill bd. nev c W&IV dm BV 28, According to put-call parity, buying a put option on a stock is equivalent to: Buying a call option and buying the stock with funds borrowed at the risk-free rate. Selling a call option and buying the stack with funds borrowed at the risk-free rate. Buying a call option, selling the stock and investing the proceeds at the risk-free rate Selling a call option, selling the stock and investing the proceeds at the risk-free rate. aoe 29. A3 month futures contract on an equity index is currently priced at USD 1000, the underlying index stocks are valued at USD 990 and pay dividends at a continuously-compounded rate of 2 percent and the current continuously compounded risk-free rate is 4 percent. The potential arbitrage profit per contract, given this set of data, is closest to UsD 10.00 Uso 7.50 usp 5.00 UsD 1.50 Copyright © 2009 Global Association of Risk Professionals 13, All rights reserved. ©OGARP hon assOcIATOn OF isk uorEIONAS 2009 FRM Practice Exams 30, Research and model projections indicate that a specific event is likely to move the CHF against the USD. While the direction of the move is highly uncertain, itis highly likely that magnitude of the ‘move will be significant. Based on this information, which of the following strategies would provide the largest economic benefit? a, Longa call option on USD/CHF and short a put option on USD/CHF with the same strike price and expiration date b, Long a call option on USD/CHF and long a put option on USD/CHF with the same strike price and expiration date ©. Short a call option on USD/CHF and long a put option on USD/CHF with the same strike price and expiration date d._ Short a call option on USD/CHF and short a put option on USD/CHF with the same strike price and expiration date 31. Initially, the call option on Big Kahuna Inc. with 90-days to maturity trades at USD 1.40. The option has a delta of 0.5739. A dealer sells 200 call option contracts and to delta-hedge the position, the dealer purchases 11,478 shares of the stock at the current market price of USD 100 per share. The following day, the prices of both the stock and the call option increase. Consequently, delta increases to 0.7040, To maintain the delta hedge, the dealer should: Purchase 2,602 shares Sell 2,602 shares Purchase 1,493 shares Sell 1,493 shares 32. Which of the following strategies creates a calendar spread? 4. Sell a call option with a certain strike price and buy a longer maturity call option with the same strike price b. Buy a call option with a certain strike price and buy a longer maturity call option with the same strike price Sella call option with a certain strike price and buy a shorter maturity call option with the same strike price 4. Buy a call option with a certain strike price and sell a longer maturity call option with the same strike price Copyright © 2009 Global Association of Risk Professionals 14 All rights reserved. OGARP 33, 35, ‘A sso Fm vores 2009 FRM Practice Exams Which of the following underlying macro-economic conditions would leave an emerging market most vulnerable to the contagion effects of a currency crisis? Large current account surplus, low foreign exchange reserves, non-convertible currency Large current account deficit, low foreign exchange reserves, fully convertible currency Small current account deficit, high foreign exchange reserves, non-convertible currency Large current account surplus, high foreign exchange reserves, fully convertible currency |. Consider an FRA (forward rate agreement) with the same maturity and compounding frequency as a Eurodollar futures contract. The FRA has a LIBOR underlying. Which of the following statements are true about the relationship between the forward rate and the futures rate? They should be exactly the same The forward rate is normally higher than the futures rate The forward rate is normally lower than the futures rate They have no fixed relationship aoce Your bank is an active player in the commodity market. The view of the economist of the bank is that inflation is expected to rise moderately in the near term and market volatility is expected to remain low. The traders are advised to undertake deals on the metals exchange to align your book to conform with the expectations of the economist of the bank. As risk manager, you are asked to monitor the positions of the traders to make sure that they have the exposures to inflation and market volatility sought by the bank, Which trader has taken an appropriate position among the traders you are monitoring? Trader A bought a call and a put, both with 90-days to expiration and with strike price equal to the existing spot level Trader B bought a put option with a down-and-in knock in feature ©. Trader C bought a call option at the existing spot levels and sold a call at a higher strike price, both with 90-days to expiration d. Trader D sold a call and bought a put at the existing levels, both with 90-days to expiration Copyright © 2009 Global Association of Risk Professionals 15, All rights reserved. @OGARP kop. assaciaTion oF wisn Paoressonats 2009 FRM Practice Exams 36. The information ratio of the Sterole US Fund for 2006 against the S&P 500, its benchmark index, is 1. For the same time period, the fund's Sharpe ratio is 2, the fund has a tracking error of 7% against the S&P 500, and the standard deviation of fund returns is 59. The risk- free rate in the US is 4%. Calculate the return for the S&P 500 during the time period. a 3.5% b. 7% c 11% d. 14% 37. Afund manager recently received a report on the performance of his portfolio over the last year. According to the report, the portfolio return is 9.3%, with a standard deviation of 13.5%, and a beta of 0.83, The risk-free rate is 3.2%, the semi-standard deviation o,(R,) of the portfolio is 8.4%, and the tracking error of the portfolio to the benchmark index is 2.8%. What is the difference between the value of the fund’s Sortino ratio (computed relative to the risk-free rate) and its Sharpe ratio? a 0.274 b. 1.727 0.653 d. 0.378 38. Which of the following statements about the linear regression of the return of a portfolio over the return of its benchmark presented below are correct? Portfolio parameter Value Beta 1.25 Alpha 0.26 Coefficient of determination 0.66 Standard deviation of error 2.42 |. The correlation is 0.71 |l, 343% of the variation in the portfolio return is explained by variation in the benchmark return Ill, The portfolio is the dependent variable IV. For an estimated portfolio return of 12%, the confidence interval at 95% is [7.16%; 16.84%] a. Wand Vv b, Mllandiv cc IMand it d. Hl, Mand iv Copyright © 2009 Global Association of Risk Professionals 15: All rights reserved. 2009 FRM Practice Exams 39. Your Board of Directors wants a comprehensive review of each business units’ operational risk activities. As the head of the corporate operational risk unit, you know that little has been done to implement an operational risk process at the business unit level and that you need to immediately come up with a framework. Which of the following statements offers the best strategy? |. The audit committee of the Board should first define its objectives to ensure that all the firm's business units’ operational risk programs are providing required information 1. The auditing department is to be charged with developing an operational risk program for each business unit, with the business unit being made clearly aware that they will be held accountable for its implementation ll, That your department immediately assess the operational risk for each business unit using. independent data feeds to ensure the information fed into the assessment cannot be manipulated IV. Asenior manager from each profit center is to be charged with developing their own operational risk self assessment program based on guidelines you provide. a, lonly b. fand1V only & tand tit only d, only 40. Which of the following risk management strategies of a firm which has principal payments to make on its debt in one year that substantially exceed the market value of its assets is most likely to be in the interest of the shareholders? a. Reduction of the overall risk level of the firm 'b. Increase of the overall risk level of the firm Keep the same risk level d._ It is impossible to say which risk management strategy the shareholders prefer END OF 2009 FRM Level | PRACTICE EXAM Copyright © 2009 Globe Association of Risk Professionals 17 Allrights reserved. ©OGARP nat asoeunio mse roresinats 2009 FRM Practice Exams This page is intentionally left blank, Copyright © 2009 Global Association of Risk Professionals 18 All rights reserved. 2 E S & a g 8 o £ a = md rs a 3 S N 2009 FRM Level | Practice Exam Answer Key Q00099000000000000O 00000000000000800000 G00O0000000000O00G0G8 00600000000000000000 00000000800800000000 All ights reserved. OGARP ‘LER ASs0EUTION OF SK PROFESSIONALS 2009 FRM Practice Exams This page is intentionally left blank. Copyright © 2009 Global Association of Risk Professionals 20, All rights reserved. 2009 FRM Practice Exams 2009 FRM Level | Practice Exam Answers & Explanations 1. To hedge against future, unanticipated, and significant increases in borrowing rates, which of the following alternatives offers the greatest flexibility for the borrower? a, Fixed for floating swap b, Interest rate collar Interest rate floor 4. Call swaption CORRECT: D The question focuses on flexible management of borrawing expenses. While a fixed for floating ‘swap could reduce borrowing expenses, itis a long-term contractual commitment to exchange Payments. if interest rates decline, the borrower may gross up to the agreed fixed rate. An interest rate collar is a combination of an interest rate floor and cap, Le, it locks in the interest expenses within a tight range. Moreover, collars usually offer interest rate protection at one particular point of time unless several contracts with different maturities are exchanged. A call swaption gives the company the right to enter into a swap when the borrowing expenses exceed a certain reference rate. If the reference rate is below the borrowing expenses, the option is not exercised. Reference: Hull, Chapter 7, 2. An investment bank uses the Exponentially Weighted Moving Average (EWMA) technique with lambda of 0.9 to model the daily volatility of a security, The current estimate of the daily volatility is 1.5%. The closing price of the security is USD 20 yesterday and USD 18 today. Using continuously- compounded returns, what is the updated estimate of the volatility? a 5.48% b. 3.62% 2.96% d. 131% CORRECT: B The current return of the security is = In (18/20) = -10.536%, Copyright © 2009 Global Association of Risk Professionals 21 Allrights reserved. OG A R E 2009 FRM Practice Exams Using an EWMA model, the updated volatility is given as: vit) flambda® ((V{t-1]42) +(1~lambda)*(current return*2)} 4 0.5 = (0.9 * ((0.015%2) + (1-0.9) * (-0,10536%2 }} 0.5 = 3.62% INCORRECT: A ~ Forgets to square the volatility terms INCORRECT: C— Forgets to square the volatility terms and to take the squore root of the resulting variance, then miscalculates conversion to percentage. INCORRECT: D— Forgets to take the square root of the variance, then miscalculates conversion to percentage. Reference : Hull, Chapter 21. 3. Consider two stocks A and B. Assume their annual returns are jointly normally distributed, the marginal distribution of each stock has mean 2% and standard deviation 10%, and the correlation is, 0.9. Whats the expected annual return of stock A if the annual return of stock B is 396? 2.9% 2% 1.1% 4.7% CORRECT: A Elite | fo =X] = Ha * (a0 %a0r/0"a)(X — py) = 0.02 + 0.9 * (0.03 - 0.02) = 0.029 Reference: Damodar Gujarati, Chap 2,3. 4. In pricing a derivative using the Monte Carlo method, we need to simulate a reasonable number of paths for the price of the underlying asset. Suppose we use a simple model for the return of the underlying asset: vit) =drift*a, + vol * v4, * e(t), and e(t) is distributed ~ N(0,1), where drift and vol are known parameters and A is the step size. Copyright © 2009 Glot Allrights reserved. | Association of Risk Professionals 22 OGARP ‘Lean ssncumon ov mse PoressonsLs 2009 FRM Practice Exams The generation of each path requires a number of steps. Which of the following describes the correct procedure? 3. Generate a random number from a normal distribution N(0,2), use the inverse normal function to get elt), which will be fed into the model to get y(t). Repeat the same procedure until you get the full desired path. b, Generate a random number from a normal distribution N(0,1), use the cumulative normal function to get e(t), which will be fed into the model to get y(t). Repeat the same procedure until you get the full desired path, ©. Generate a random number from a uniform distribution defined in [0,1], use the inverse cumulative normal function to get e(t), which will be fed into the model to get y(t). Repeat the same procedure until you get the full desired path, d. Generate a random number from a uniform distribution defined in [0,1], use the cumulative normal function to get e(t), which will be fed into the madel to get yit). Repeat the same procedure until you get the full desired path. CORRECT: C This question wants to test if the candidate knows the basic steps to generate a very simple path: ‘answering this question means that the candidate would be able to build a simple spreadsheet ‘showing the Monte Carla lagic, The correct procedure is the one described in c); while a), b) and d) are nonsensical calculations. Reference: Philippe Jorion, Value at Risk, The New Benchmark for Managing Financial Risk, 3 edition (New York: McGraw-Hill, 2007), Chapter 12. 5. Arisk manager estimates the daily variance (hi) using a GARCH model on daily returns (r): hi= 00+ crea + Bi Assume the model parameter values are cu) = 0.005, a4 = volatility is approximately: 9.04, B = 0.94. The long-run annualized a. 25.00% b. 13.54% c 72,72% d. 7.94% CORRECT: D Copyright © 2009 Global Association of Risk Professionals 23, Allrights reserved. © 2009 FRM Practice Exams The long-run variance is 0.005/(1-0.04-0.94) =0.005/0.02 = 0.25. The daily vol s thus the square root, or 0.5% and annual vol 7.935%, INCORRECT: A ~The daily variance is indeed 0.25%, and the daily volatility 0.5% but this needs to be ‘annualized, INCORRECT: B ~ Miscalculates variance as sqrt(0,04/(1 ~ 0.94 —0.005)) * 15.87 = 13.54% INCORRECT: C- Miscalculates variance as 0.04/(1 0.94 0.005) = 72.7296 Reference: Hull, Chapter 21. 6, Asingle stock has a price of USD 10 and a current dally volatility of 2%. Using the delta-normal ‘method, the VaR at the 95% confidence level of a long at-the-money call on this stock over a 1-day holding period is approximately: usp 0.23 USD 1.645 sD 0.33 uso 0.16 CORRECT: D This question requires candidates to know the formula for the delta-normal VaR approximation, and «also to know that the delta of an at-the-money call is 0.5. VaR =| A|x1.645 xox S = 0.5x1,645 x 0.02 x10 = 0.1645. INCORRECT: A~We get A by using 2.326 instead of 1.645. INCORRECT: 8~ We get B if we use 2 instead of 2% for the volatility. INCORRECT: C— We get Cif we use a delta of 1. Reference; Allen et al, Chapter 3, 86-89 7. Aportfolio consists of two zero coupon bonds, each with a current value of USD 10. The first bond has a modified duration of 1 year and the second has a modified duration of 9 years. The yield curve is flat and all yields are 5%. Assume all moves of the yield curve are parallel shifts. Given that the daily volatility of the yield is 1%, which of the following is the best estimate of the portfolio daily VaR at the 95% confidence level? ‘Copyright © 2009 Global Association of Risk Professionals 24 All rights reserved. OGARP ana associ orn resins 2009 FRM Practice Exams USD 2.33 USD 1.65 USD 0.82 USD 1.16 CORRECT: B This question assesses candidates’ abilities to apply the duration VaR formula to two bonds simultaneously and to recall that the duration of a zero coupon bond is equal to the bond maturity. Using an obvious extension of Jorion’s equation 9.5 VaR = D, xV,x1.645x 0 + D, xV, x1.645x0 = (D, xV, + D, x Vy) x1.645x0 = (D, + D,)x10%1,645xo = 10*101.645% 0,01 = 1.645 INCORRECT: A ~The 9936 confidence level VaR INCORRECT: C- Arises if the candidate mistakenly divides the correct answer by the number of bond INCORRECT: D ~ Makes both mistakes Reference: Allen et al. 8. Consider the following three methods of estimating the P&L of a bullet bond: full repricing, duration {PV02), and duration plus convexity. Ranking the estimated P&L impact of a large negative yield shock from the lowest P&L impact to the highest P&L impact, what is the ranking of the methods to estimate the P&L impact? @. Duration plus convexity, duration, full repricing b. Fullrepricing, duration plus convexity, duration c. 4. Duration, duration plus convexity, full repricing Duration, full repricing, duration plus convexity CORRECT: C The price / yield line with yield on the x axis and price on the y axis is convex to the origin. The duration at any yield level is the tangent to that curve. Therefore, except at the exact point of tangency, duration will always underestimate the price change. INCORRECT: A~ Duration will always underestimate price change for negative yield shacks INCORRECT: 8 ~ Full repricing will never generate a smaller positive price change than duration because duration represents the point of tangency INCORRECT: D ~ Full repricing wil generate a higher price for a large negative yield change than will duration plus convexity Reference: Allen, Boudoukh, Saunders, Chapter 3 Copyright © 2009 Global Association of Risk Professionals 25 Allrights reserved. @OGARP 2009 FRM Practice Exams 9. Consider a position in a 5-year receive-fixed swap that makes annual payments on a USD 100 million notional. The floating leg has just been reset. The term structure is flat at 5%, the Macaulay duration of a 5-year par bond is 4.5 years, and the annual volatility of yield changes is 100bp. Your best estimate of the swap’s VaR with 95% confidence over the next month is USD 1.6 million USD 2.0 million USD 5.5 million USD 7.1 million aoop CORRECT: A Because the floating-rate leg has just been reset, its duration is 1. Net duration is 4.5-1=3.5 year, or ‘modified duration of 3.5/1.05=3.33. The 95% VaR of monthly changes in yields is 1.65%1%/ V1.2 = 0.48%. Multiplying, this gives USD 100*0.48%*3.33=USD 1.588 INCORRECT: 8 ~ This uses a net duration of 4.5 years and ignores the duration of the floating-rate leg. INCORRECT: C- This is the annual VaR, but should be translated to a monthly horizon. INCORRECT: D = This is the annual VaR computed by ignoring the duration of the floating-rate leq Reference: Allen et al. 10. Ifthe gold lease rate is higher than the risk-free rate, what is the market structure of the forward market for gold? a. Contango b. Backwardation c. Inversion d. Need more information to determine CORRECT: B Alease rate higher than the risk fee rate will force a negatively sloped forward curve, i.e. backwardation INCORRECT: A ~ The forward price = spot*exp( risk free rate - lease rate). if the lease rate is higher than the risk free rate, forwards will be lower than spot, implying contango INCORRECT: C— The term inversion is used to describe yield curves, not commodity forwards INCORRECT: D ~ There is enough information in the question to provide an answer Reference: MacDonald, Chapter 6 Copyright © 2009 Global Association of Risk Professionals 26, All ights reserved. OGARP ona Asoc oF mae orEsonas 2009 FRM Practice Exams 11. The price of a 3-year zero coupon government bond is 85.16. The price of a similar 4-year bond is 79.81, What is the one-year implied forward rate from year 3 to year 4? 5.4% 5.5% 5.8% 6.7% CORRECT: D Price of three bond _ 85.16 14 Forward rate = Price of three bond __ 85.16, Price of four year bond 79.81 1.087034 Forward rate = 0.067034 or 6.7% INCORRECT: A ~ This 's B/C INCORRECT: B ~ This is the return of the 3-year bond INCORRECT: C— This is the return of the 4-year bond Reference: Tuckman 12. Aportfolio manager has a bond position worth USD 100 million. The position has a modified duration of 8 years and a convexity of 150 years. Assume that the term structure is flat. By how much does the value of the position change if interest rates increase by 25 basis points? usp -1,953,125 USD -1,906,250 USD -2,046,875 USD -2,187,500 CORRECT: A AV =-Daoy * Ay V + 0.5 Convexityx Ay? x V AV =-80.0025x100M +0.5 x150x(0.0025)* x100M AV =-2M +46,875 AV =-1,953,125, INCORRECT: B~ Omits 0.5 from the second term INCORRECT: C—Subtracts the second term INCORRECT: D-Makes both mistakes Reference: Tuckman Copyright © 2009 Global Association of Risk Professionals 27 All ights reserved, ©OGARP ‘ROBAL AMOELTION OF IS HOFESIONAS 2009 FRM Practice Exams 413. A firm is going to buy 10,000 barrels of West Texas Crude Oil. It plans to hedge the purchase using the Brent Crude futures contract. The correlation between the spot and futures prices is 0.72. The volatility of the spot price is 0.35 per year. The volatility of the Brent Crude futures price is 0.27 per year. What is the hedge ratio for the firm? a. 05554 b. 09333 c. 1.2099 d, 08198 CORRECT: B 0.35) re( S27) N-0.9993 INCORRECT: A — inverts the spot volatility and the futures volatility INCORRECT: C~ Uses variances INCORRECT: D — Uses square roots of the volatilities Reference: Hull, Chapter 3 14. Its June 2” and a fund manager with USD 10 million invested in government bonds is concerned that interest rates will be highly volatile over the next three months. The manager decides to use the September Treasury bond futures contract to hedge the value of the portfolio, The current futures price is 95.0625. Each contract is for the delivery of USD 100,000 face value of bonds. The duration of the manager's bond portfolio in three months will be 7.8 years. The cheapest to deliver bond in the Treasury bond futures contract is expected to have a duration of 8.4 years at maturity of the contract. At the maturity of the Treasury bond futures contract, the duration of the underlying benchmark Treasury bond is 9 years. What position should the fund manager undertake to mitigate his interest rate risk exposure? a. Short 94 contracts b. Short 98 contracts Short 105 contracts 6. Short 113 contracts CORRECT: B. Copyright © 2009 Glok All rights reserved. | Association of Risk Professionals 28 2009 FRM Practice Exams 78 »-(corease (Es) N=97.68 or 98 contracts = This is made up. = This leaves out the durations INCORRECT: D This inverts the durations Reference: Tuckman 15. Abond trader has bought a position in Treasury Bonds with a 4% annual coupon rate on February 15, 2015. The DVO1 of the position is USD 80,000. The trader decides to hedge his interest rate risk with the 4.5% coupon rate Treasury Bonds maturing on May 15, 2017 which has a DVO1 of .076 per USD 100 face value. To implement this hedge, approximately what face amount of the 4.5% ‘Treasury bonds maturing on May 15, 2017 should the trader sell? USD 80,000 USD 10,500,000 USD 80,000,000 USD 105,000,000 ‘CORRECT: D USD 105,000,000x.076/100 = USD 79,800, which is pretty close to the desired DVO1 of USD 80,000, To solve for the hedge, solve for F in the equation USD 80,000 = Fx.076/100, giving F = 105,263,158 INCORRECT: A ~ Selling this amount would offset a DVO1 of only USD 80,000x.076/100 = USD 61 INCORRECT: 8 ~ USD 10,500,000x.076/100 = USD 7,980 INCORRECT: C~ USD 80,000,000x.076/100 = USD 60,800 Reference: Tuckman, Chapter 5 Copyright © 2009 Global Association of Risk Professionals 29, All ights reserved. OGARP SLL ASEIATON OF Re FOFESINALS 2009 FRM Practice Exams 16, Suppose that A and B are random variables, each follows a standard normal distribution, and the covariance between A and B is 0.35. What is the variance of (3A + 28)? a, 15.10 b, 14.47 «9.20 d, 17.20 CORRECT: D Since each variable is standardized, its variance is 1. Therefore V(3A+2B) = 9 V(A) + 4 V(B)+2x 3x2 x Cov(A,B) = 94444,2 = 17.2 INCORRECT: A~ 9+4+6*0,35=15.1 INCORRECT: B= 9+4+12*0.35*2= INCORRECT: C~ 3+2+12*0,35=9,2 Reference: Damodar Gujarati 17. Consider a stock price S that follows a geometric Brownian motion dS = 1 S dt + B S dz, with strictly positive and ji a fixed value. Which of the following statements is true? if the drift 1 is negative, the price one year from now will be below today’s price. ‘The instantaneous rate of return on the stock follows a uniform distribution, ‘The stock price S follows a lognormal distribution. This model imposes mean reversion. CORRECT: C INCORRECT: A ~The expected price is less than today’s price, but not the price in all the states of world, INCORRECT: B~ The instantaneous rate of return on the stock follows normal distribution. INCORRECT: D-This model does not impose mean reversion. Reference: Philippe Jorion, Value at Risk: The New Benchmark for Managing Financial Risk, 3" ed, (New York: McGraw-Hill, 2007). Chapter 12 Copyright © 2009 Global Association of Risk Professionals 30. All rights reserved. 2009 FRM Practice Exams 18, The joint probability distribution of random variables X and Y is given by f(xy) = kxy for x= 1, 2, 3, y= 1, 2, 3, and kis a positive constant. What is the probability that X + Y will exceed 5? a. 1/9 b 1/4 c 1/36 d. Cannot be determined CORRECT: B ae Note that 1 > f(x,» Substituting the various values of x and y, we get fit,1)=k,f(2,2)=2k,f(1,3)=3k, fl2,1}=2k, f(2,2)=4k, S12,3)-6k, f(3,1)=3k, (3,2) Reference: Damodar Gujarati 19. Which of the following statements regarding Hypothesis Testing is incorrect? ‘a. Hypothesis testing is used to make inferences about the parameters of a given population on the basis of statistics computed for a sample that is drawn from that population. Type ll error refers to the failure to reject the null hypothesis when it is actually false. ©The p-value decision rule is to reject the null hypothesis if the p-value is greater than the significance level. d, Allelse being equal, the decrease in the chance of making a Type | error comes at the cost of increasing the probability of making a Type Il error. CORRECT: C The true statement is to reject Ho If the p-value is smaller than the significance level. INCORRECT: A —Statement A is correct regarding the primary use of Hypothesis Testing. INCORRECT: 8 ~ Statement B is correct regarding the definition of type il error. INCORRECT: D ~ Statement D is correct because type | error and type Il error are in tradeoff. Reference: Damodar Gujarati Copyright © 2009 Global Association of Risk Professionals 31 All rights reserved. OG ARP 2009 FRM Practice Exams 20, If stock returns are independently identically normally distributed and the annual volatility is 30%, then the daily VaR at the 99% confidence level of a stock market portfolio is approximately: 2.41% 3.11% 4.40% 1.89% CORRECT: C The 1-day volatility is 5 * (1/252)40.5 = 0.3 * 0.629941 = 0.018898. The VaR at the 99% confidence level is then equal to 2.32635 * 0.018898 = 4.40% INCORRECT: A~One gets A if one uses 1.645 instead of 2.326 INCORRECT: 8 - One gets 8 if one uses the monthly volatility instead of the daily one INCORRECT: D~ One gets D is the daily volatility Reference: Allen, Boudoukh and Saunders, 2004, chapter 1, p 6-8 21, The current value of the S&P 500 index is 1457, and each S&P futures contract is for delivery of USD 250 times the index. A long-only equity portfolio with market value of USD 300,100,000 has beta of 1.1, To reduce the portfolio beta to 0.75, haw many S&P futures contract should you sell? a, 618contracts 288 contracts . 574contracts d. 906 contracts CORRECT: 8 No of contracts = [0.75 —1.1)/ 1}* [300,100,000 / {250 * 1,457}] =-288.36 sell 288 contracts INCORRECT: A~ -617.9135209 = -1*(0.75)* (300100000 / (250*1457)) INCORRECT: C— -561.74 = -1(0.75/1.1)* (300100000 /(250*1457)) INCORRECT: D~ -906.273164 = -1* (1.1)* (300100000 / (250*1457)) Reference: Hull, Options, Futures and Other Derivatives, Chapter 3 and 4; Anthony Saunders, Financial institutions Management, Chapter 10 Copyright © 2009 Global Association of Risk Professionals 32 All rights reserved. OGARP -QOBAL ASSOCIATION OF WK PROFESSIONALS 2009 FRM Practice Exams ‘The following information should be used for the next two questions. (On January 4, a risk manager observes that the 1-year continuously compounded interest rate is 5% and storage costs of a commodity product A is USD 0.05 per quarter (payable at each quarter end), He further observes the following forward prices for product A: March 5.35 June 5.90 September 5.30 December 5.22 22. Given the following explanation of supply and demand for commodity product A how would you best describe its forward price curve from June to December? Market description Explanation a. Backwardation Excess demand for A in early summer b. Backwardation ‘Supply is expected to decline after summer c. Contango Excess demand for A in early summer d. Contango ‘Supply is expected to decline after summer CORRECT: A Ais correct - when further-term commodity forwards have lower price than near-term forwards, the market is said to be in ‘backwardation’. Possible explanation can be seasonality of product A ~ excess demand in early summer causes June forwards to have higher price INCORRECT: B~ Market description is correct, but explanation is not ~ expected decline in supply should increase further term commodity forward price — Wrong market description of contango ~ Wrong market description of contango Reference: Robert L McDonald, Derivatives Markets, Chapter 6 23. What is the annualized rate of return earned on a cash-and-carry trade entered into in March and closed out in June? 8.9% 9.8% 35.7% 39.1% ange CORRECT: C Copyright © 2009 Global Association of Risk Professionals 33, Allrights reserved. 2009 FRM Practice Exams By formula Fo,r = Soe + C, where For = June forward price, Sp = March forward price, r = risk ‘free interest rate, T= length of cash-and-carry, C= storage cost Solving 5.90 = 5.35€”? + 0.05 Solution is r = 35.73% INCORRECT: A 8.9 = LN((5.9-0.05)/5.35) (forgets to annualize the return) INCORRECT: B— 9.8=LN((5.9)/5.35) (forgets to include the storage cost and to annualize the return) INCORRECT: D ~ 39.1= (12/3)LN({5.9)/5.35) ~ 0.05 [forgets to include the storage cost) Reference: Robert L McDonald, Derivatives Markets, Chapter 6 24. An investor sells a June 2008 call of ABC Limited with a strike price of USD 45 for USD 3 and buys a June 2008 call of ABC Limited with a strike price of USD 40 for USD 5. What is the name of this strategy and the maximum profit and loss the investor could incur? a. Bear Spread, Maximum Loss USD 2, Maximum Profit USD 3 b, Bull Spread, Maximum Loss Unlimited, Maximum Profit USD 3 ©. Bear Spread, Maximum Loss USD 2, Maximum Profit Unlimited d. Bull Spread, Maximum Loss USD 2, Maximum Profit USD 3 CORRE Buying a call option at lower stock price and selling call option at higher strike price is called as Bull Spread. Bear Spread is buying the call option at higher price and selling the call at lower strike price. The Cost of strategy will be USD 3-USD 5 = -USD 2 The Payoff, when Stock price Sr USD 40 will be -USD 2 (the cost of strategy) as none of the option will be exercised. The Payoff, when stock price $;2 45, (as both options will be exercise) will be USD 5, Since the cost of strategy is USD 3, hence profit will be USD 5-USD 2 = USD 3 When Stock price is USD 40< S;> USD 45, Only the call option bought by the investor would be exercised hence the pay off will be Sr~40, since the cost of strategy is -USD 3, The Net profit will be $S:~43, which would always be lower than USD 3. Reference: Hull. Chapter 10- Trading Strategies Involving Options. Copyright © 2009 Global Association of Risk Professionals 34 Allrights reserved. OG ARP 2009 FRM Prac' e Exams 25. Which of the following problems are NOT inherent disadvantages of the historical simulation approach to estimating VaR? |. Itgives too little weight to more recent observations I. For long-only portfolios, itis likely to understate VaR following a recent structural increase in volatilities lll. Ttalways ignores the fat tails present in the distribution of returns on many financial assets IV. Because of the delta approximation, it inadequately measures the risk of nonlinear instruments Vand itonly only |, land IV only Wand IV only aoge CORRECT: C The disadvantage with the Historical Simulation Model is that it may not recognize the changes in volatility and correlation following recent structural changes. The model can be adjusted so that it gives more weight to recent observations. The other options, i.e. Ill & IV, are disadvantages of Monte Carlo method and Delta-normal method. Reference: Allen et al. 26. A bank holds USD 60 million worth of 10-year 6.5% coupon bonds that are trading at a clean price of 101.82. The bank is worried by the exposure due to these bonds but cannot unwind the position for fear of upsetting the client. Therefore, it purchases a total return swap (TRS) in which it receives annual Libor + 100 bps in return for the mark-to-market return on the bond. For the first year, the Libor sets at 6.25% and by the end of the year the clean price of the bonds is at 99.35. The net receipt/payment for the bank in the total return swap will be: ‘a. Receive USD 2.23 million b. Receive USD 1.93 million, Pay USD 1.93 million. dd. Pay USD 2.23 million. CORRECT: B Copyright © 2009 Glot All ights reserved. ‘Association of Risk Professionals 35 2009 FRM Practice Exams It's the result of this calculation: the notional amount is 60 million USD . Therefore the bank will receive the interest payment linked to the LIBOR rate: 60 million USD * (6,25%+100 bp) = 4. 35 million USD The bank will pay the fixed coupon plus the change in the value of the bond: 60 million USD * 6.5% + 60 million *(99,35%-101,82%) = 2.418 million USD. Hence the total net amount the bank will receive is: 4,35 million USD - 2.418 million USD = 1.932 ‘itlion USD Reference: Hull, Chapter 7 27. Which of the following trade(s) contain basis risk? \ Long 1,000 lots Nov 07 ICE Brent Oil contracts and short 1,000 lots Nov 07 NYMEX WTI Crude Oil contracts " Long 1,000 lots Nov 07 ICE Brent Oil contracts and long 2,000 lots Nov 07 ICE Brent Oil at- the-money put Ml, Long 1,000 lots Nov 07 ICE Brent Oil contracts and short 1,000 lots Dec 07 ICE Brent Oil contracts IV. Long 1,000 lots Nov 07 ICE Brent Gil contracts and short 1,000 lots Dec 07 NYMEX WTI Crude Oil contracts a 1@ll b. W&IV ce M&IV dN & IV CORRECT: D Basis Risk is spread risk, which arise from trading the spread (long and short 2 positively correlated assets or same asset with different expiration) iis spread trade in highly correlated asset with same expiration month fi faces with gamma and vega risk itis spread trade in trading the flattening of the forward curve ivis spread trade in trading 2 assets with different expiration date Reference: Robert |. McDonald, Derivatives Markets (Boston: Addison-Wesley, 2003), Chapter 6. Copyright © 2009 Global Association of Risk Professionals 36 Allrights reserved. 2009 FRM Practice Exams 28, According to put-call parity, buying a put option on a stock is equivalent to: Buying @ call option and buying the stock with funds borrowed at the risk-free rate. Selling a call option and buying the stock with funds borrowed at the risk-free rate. Buying a call option, selling the stock and investing the proceeds at the risk-free rate. Selling a call option, selling the stock and investing the proceeds at the risk-free rate. CORRECT: C ‘Buying a call option, selling the stock and investing the proceeds at the risk-free rate. Put-call parity states P=C-S+Xe"” INCORRECT: A ~ Buying a call option is correct, but the rest of the statement is Incorrect. INCORRECT: B - The entire statement is incorrect. INCORRECT: D— Selling a call option is incorrect, but the rest of the statement is correct. Reference: Hull, Chapter 10 29, A3 month futures contract on an equity index is currently priced at USD 1000, the underlying index stocks are valued at USO 990 and pay dividends at a continuously-compounded rate of 2 percent and the current continuously compounded risk-free rate is 4 percent. The potential arbitrage profit er contract, given this set of data, is closest to a. USD 10.00 b. USD 7.50 «. usD5.00 d. usp 1.50 CORRECT: C According to the fundamental pricing relationship between spot assets and the associated futures, the futures price, to prevent arbitrage, should equal 990 x e (0.04 — 0.02) x 0.25 or 995. Hence, the {futures contract is overvalued, indicating it should be sold and the index should be purchased for an arbitrage profit of USD 1000 - USD 995 = USD 5 Reference: Hull, Chapters 2,3 6, Copyright © 2009 Global Association of Risk Professionals 37 All rights reserved. 2009 FRM Practice Exams 30. Research and model projections indicate that a specific event is likely to move the CHF against the USD. While the direction of the move is highly uncertain, it is highly likely that magnitude of the ‘move will be significant. Based on this information, which of the following strategies would provide the largest economic benefit? a. Longa call option on USD/CHF and short a put option on USD/CHF with the same strike price and expiration date b. Longa call option on USD/CHF and long a put option on USD/CHF with the same strike price and expiration date Short a call option on USD/CHF and long a put option on USD/CHF with the same strike price and expiration date 4. Short a call option on USD/CHF and short a put option on USD/CHF with the same strike price and expiration date CORRECT: B The question tests on understanding of a “straddle” strategy and its application on currency trading. Along straddle strategy involves buying (long) a call and put option with the same strike price and expiration date, and will benefit most when the underlying moves away from the current equlibrium. INCORRECT: A — it sells a put option while it should buy one put INCORRECT: C—It sells a caill option while it should buy one call INCORRECT: D—/t sells both the call and put option while it should buy both Reference: Hull, Chapter 10, 31. Initially, the call option on Big Kahuna Inc, with 90-days to maturity trades at USD 1.40. The option has a delta of 0.5739. A dealer sells 200 call option contracts and to delta-hedge the position, the dealer purchases 11,478 shares of the stock at the current market price of USD 100 per share. The following day, the prices of both the stock and the call option increase. Consequently, delta increases to 0.7040. To maintain the delta hedge, the dealer should: a. Purchase 2,602 shares. b. Sell 2,602 shares. ©. Purchase 1,493 shares. 4. Sell 1,493 shares. CORRECT: A ‘Number of calls = 200 contracts x 100 = 20,000 calls. Copyright © 2009 Global Association of Risk Professionals 38 All rights reserved. © G A R 2009 FRM Practice Exams Number of shares = (Number of calls) x (New delta ~ Old delta) = 20,000 x (0.7040 -0.5739) = +2,602 shares Positive sign indicates that the manager should purchase new shares. INCORRECT: 8 ~ The formula is incorrect, i. old delta minus new delta INCORRECT: C~ The number of shares (instead of number of calls) is used in the calculation INCORRECT: DAs per explanation in ‘C’ above and sign error Reference: Hull Chapters 9 and 10 32. Which of the following strategies creates a calendar spread? a, Sell a call option with a certain strike price and buy a longer maturity call option with the same strike price b. Buy call option with a certain strike price and buy a longer maturity call option with the same strike price Sella call option with a certain strike price and buy a shorter maturity call option with the same strike price 4. Buy call option with a certain strike price and sell a longer maturity call option with the same strike price CORRECT: A INCORRECT: B~As buy a.call option INCORRECT: C—As buy a shorter-maturity call option INCORRECT: D ~ As this is @ reverse calendar spread Reference: John Hull, Chapter 10. 33. Which of the following underlying macro-economic conditions would leave an emerging market most vulnerable to the contagion effects of a currency crisis? a, Large current account surplus, low foreign exchange reserves, non-convertible currency b, Large current account deficit, low foreign exchange reserves, fully convertible currency ©, Small current account deficit, high foreign exchange reserves, non-convertible currency d._ Large current account surplus, high foreign exchange reserves, fully convertible currency CORRECT: 8 Copyright © 2009 Global Association of Risk Professionals 39, All rights reserved. OGARP ‘Lob As9caTION OF NK MrESIONS 2009 FRM Practice Exams INCORRECT: A ~Large current account surplus and non-convertible currency would protect the local currency INCORRECT: C— High foreign exchange reserves and non-convertible currency would protect the local currency INCORRECT: D ~ Large current account surplus and high foreign exchange would protect the local currency Reference: Saunders, Chapter 15, Foreign Exchange Risk 34. Consider an FRA (forward rate agreement) with the same maturity and compounding frequency as @ Eurodollar futures contract. The FRA has a LIOR underlying. Which of the following statements are true about the relationship between the forward rate and the futures rate? a. They should be exactly the same b. The forward rate is normally higher than the futures rate The forward rate is normally lower than the futures rate d._ They have no fixed relationship CORRECT: C As Eurodollar futures contract is marked to market and settled daily, normally forward rate is adjusted lower, so called convexity adjustment, by: Forward rate = Futures rate — fo? RY, Reference: Hull, Chapter 6. 35. Your bank is an active player in the commodity market. The view of the economist of the bank is that inflation is expected to rise moderately in the near term and market volatility is expected to remain low. The traders are advised to undertake deals on the metals exchange to align your book to conform with the expectations of the economist of the bank. As risk manager, you are asked to monitor the positions of the traders to make sure that they have the exposures to inflation and market volatility sought by the bank. Which trader has taken an appropriate position among the traders you are monitoring? a. Trader A bought a call and a put, both with 90-days to expiration and with strike price equal to the existing spot level b. Trader B bought a put option with a down-and-in knock in feature c. Trader C bought a call option at the existing spot levels and sold a call at a higher strike price, both with 90-days to expiration d. Trader D sold a call and bought a put at the existing levels, both with 90-days to expiration Copyright © 2009 Global Association of Risk Professionals 40, Allrights reserved. © G A R P 2009 FRM Practice Exams CORRECT: C Cis correct, as the strategy popularly known as the bull spread will result in positive payoff when the spot rises. As inflation increases, spot levels in commodities are expected to rise. Selling a call at higher level will reduce the cost of the strategy. Although it may limit the upside, but that would be in line with the view as only a moderate rise is expected in spot. INCORRECT: A~ is incorrect, as the strategy popularly known as a straddie is tobe used when the view is that the volatility in the market will rise, and there is no directional view on the spot INCORRECT: B Is incorrect, as the above option will be suitable when the spot is expected to fall {from the existing levels INCORRECT: D—(s incorrect, as the payoffin this case is similar to short position in spot and would make sense when the underlying is expected to fall Reference: Hull, Chapter 10. 36. The information ratio of the Sterole US Fund for 2006 against the S&P 500, its benchmark index, is 1. For the same time period, the fund's Sharpe ratio is 2, the fund has a tracking error of 7% against. the S&P 500, and the standard deviation of fund returns is 5%. The risk: free rate in the US is 4%, Calculate the return for the S&P 500 during the time period. 3.5% 1% 11% 14% CORRECT: 8 Sharpe Ratio = 2 (Fund Return — Risk Free Rate)/SD = 2 (Fund Return ~ 4%6)/5% = 2 Fund Return = 14% Information Ratio = 1 (Fund Return ~ S&P 500 Return)/ Tracking Error = 1 (149% - S&P 500 Return) / 7% S&P 500 Return = 7% INCORRECT: A ~ Incorrectly divides S&P 500 Return by 2, INCORRECT: C— The candidate might use the Tracking Error as the Numerator in both the Ratios Copyright © 2009 Global Association of Risk Professionals 41 All rights reserved. OGARP ‘SLOBAL ASSOCIATION OF SK onESIOMAS 2009 FRM Practice Exams ‘Sharpe Ratio = (Fund Return ~ Risk Free Rate)/Tracking Error = 2 (Fund Return ~ 4%)/7% = 2 Fund Return = 18% Information Ratio =1 (Fund Return - S&P 500 Return)/ Tracking Error = 1 (189 - S&P 500 Return) / 796 =1 SBP 500 Return = 11% INCORRECT: D ~The candidate can stop with the fund return calculation, and end up with 14% Sharpe Ratio = (Fund Return ~ Risk Free Rate)/SD = 2 (Fund Return ~ 4%)/5% = 2 Fund Return = 14% Reference: Amenc and Le Sourd, Portfolio Theory and Performance Analysis. Chapter 4 37. A fund manager recently received a report on the performance of his portfolio over the last year. According to the report, the portfolio return is 9.3%, with a standard deviation of 13.5%, and a beta of 0.83. The risk-free rate Is 3.2%, the semi-standard deviation o(R,) of the portfolio is 8.4%, and the tracking error of the portfolio to the benchmark index is 2.896, What is the difference between the value of the fund’s Sortino ratio (computed relative to the risk-free rate) and its Sharpe ratio? a. 0.274 b. 1.727 c. 0.653 4. -0.378 CORRECT: A R,-Rp _ 9.3% -3.2% Sharpe ratio equals to = 0.452 o(R,) 13.5% R,~R; %o-3.2% While Sortine ratio equals vo 2 = = 23% 3.2% _ 9 196 (Rp) 8.4% Tracking error is used to calculate the value of the information ratio, which is defined os R,—Ra —* "the calculation of information ratio is not required in this question. o(Rp— Ry) 0.726-0.452=0.274 INCORRECT: B~ 2.178~0.452=1.727 Copyright © 2009 Giot Allrights reserved, | Association of RiskProfessionals, 42 OG A R P 2009 FRM Practice Exams INCORRECT: C~ 0.726 ~ 0.0.73 = 0.653 (0.073 INCORRECT: D ~ 0.73-0.452=0.378 ,093 ~ 0.032)/0.83 Reference: Amenc and Le Sourd, Portfolio Theory and Performance Analysis. Chapter 4 38. Which of the following statements about the linear regression of the return of a portfolio over the return of its benchmark presented below are correct? Portfolio parameter Value Beta 1.25 ‘Alpha 0.26 Coefficient of determination 0.66 Standard deviation of error 2.42 |. The correlation is 0.71 | 3496 of the variation in the portfolio return is explained by variation in the benchmark return lil, The portfolio is the dependent variable V. Foran estimated portfolio return of 12%, the confidence interval at 95% is [7.1634;16.84%] a. land iV b. IMland iv cc. Iplland ith di tllandiv CORRECT: 8 The portfolio return is the dependent variable and for an estimated portfolio return of 12%, the 959% confidence interval is [129% - 2 * 2.42%, 12% + 2 * 2.42%] or [7.16%, 16.84%]. However, the correlation is the square root of the coefficient of determination and is therefore equal to 0.81, and 66% of the variation in the portfolio returns is explained by variation in the benchmark return. Reference: Amenc and Le Sourd, Portfolio Theory and Performance Analysis. Chapter 4 Copyright © 2009 Global Association of Risk Professionals 43, All rights reserved. ©GARP 2009 FRM Practice Exams 39, Your Board of Directors wants a comprehensive review of each business units’ operational risk activities. As the head of the corporate operational risk unit, you know that little has been done to implement an operational risk process at the business unit level and that you need to immediately come up with a framework. Which of the following statements offers the best strategy? | The audit committee of the Board should first define its objectives to ensure that all the firm’s business units’ operational risk programs are providing required information |The auditing department is to be charged with developing an operational risk program for each business unit, with the business unit being made clearly aware that they will be held accountable for its implementation I. That your department immediately assess the operational risk for each business unit using independent data feeds to ensure the information fed into the assessment cannot be manipulated IV. Asenior manager from each profit center is to be charged with developing their own operational risk self assessment program based on guidelines you provide. a. fonly b. and iV only ¢. Jandill only 4. Wonly CORRE The best strategy for developing an operational risk framework is to empower business units with the responsibility, accountability and authority to manage their own operational risks. The business units know their risks the best. INCORRECT: A~ "t's not the responsibility of the Audit Committee of the Board INCORRECT: B- The auditing department is not the best assessor of an individual business unit's risk, In fact many audit stoff do not fully understand the risks of many of a firm's activities INCORRECT: C— il’ is duplicative and should not come from the corporate department Reference: Risk Management and Copital Adequacy, Gatlati, 2003. 40. Which of the following risk management strategies of a firm which has principal payments to make ‘on its debt in one year that substantially exceed the market value of its assets is most likely to be in the interest of the shareholders? a. Reduction of the overall risk level of the firm b, Increase of the overall risk level of the firm ©. Keep the same risk level d, Its impossible to say which risk management strategy the shareholders prefer Copyright © 2009 Global Association of Risk Professionals 44 All rights reserved, OGARP 1984 ss0cumTON OF is porEssionns 2009 FRM Practice Exams CORRECT: 8 ‘Once a firm is in distress, itis not in the interests of shareholders to reduce risk. if the firm stays in distress and eventually defaults, shareholders will end up with worthless shares. In these circumstances, management intent on maximizing shareholder value will seek out new risks. Reference: Risk Management and Derivatives, Stulz, 2003 END OF 2009 FRM Level | PRACTICE EXAM Questions & Explanations Copyright © 2009 Global Association of Risk Professionals 45, All rights reserved, OGARP Ui ssoeunon ns restos 2009 FRM Practice Exams This page is intentionally left blank. Copyright © 2009 Global Association of Risk Professionals 46 Allrights reserved. 2 Ee S x a 8 3 £ a = & c a 3 Ss Aq 2009 FRM Full Exam Practice Exam | Candidate Answer Sheet GO00999090090909HHOOHOOOO CICICICICICICICICICICICICICICICICICICICICICICICIO) 6066000000000 00000000 9000000000000000000000000 RS Sa sis see sags sogeegsgesgs Q0000090990990009000000008 0000O00000000000OO0OO00000 0000660000000000000000008 0000000000000000000000000 a7 Copyright © 2009 Global Association of Risk Professionals All rights reserved. OGARP ‘SLOBL ASSECIATON OF ISK HOFESSIOMAS 2009 FRM Practice Exams This page is intentionally left blank. Copyright © 2009 Global Association of Risk Professionals 48, All rights reserved. OG A R P 2009 FRM Practice Exams 2009 FRM Full Exam Practice Exam | Questions 1. Given the information provided in the table below, what is the portfolio VaR, at the 99% confidence level, of the following 100 million CHF equally weighted investment portfolio? Expected Asset Sen Volatility | Correlation Stocks Bonds Stocks | 24.00% 18% 1 Bonds | 15.00% 6% O14 1 27.36 million CHF 22.77 million CHF 20.97 million CHF 13,98 million CHE aoge 2. You are asked by your boss to estimate the exposure of a hedge fund to the S&P 500, Though the fund claims to mark to market weekly, it does not do so and marks to market once a month. The fund also does not tell investors that it simply holds an ETF which is indexed to the S&P500. Because of the claims of the hedge fund, you decide to estimate the market exposure by regressing weekly returns of the fund on the weekly return of the S&PSO0. Which of the following properties correctly describes a property of your regression estimates? a, The beta of your regression will be one because the fund holds the S&P 500, b. The beta of your regression will be zero because the fund returns are not synchronous with the S&P 500 returns. The intercept of your regression will be positive, showing that the fund has a positive alpha when estimated using an OLS regression. 4. The beta will be misestimated because hedge fund exposures are non-linear. Copyright © 2009 Global Association of Risk Professionals 49 All rights reserved. ©OGARP 2009 FRM Practice Exams 3. The following table shows the composition of the GARP Bond Fund. What are the portfolio duration and portfolio yield of the fund? (GARP Bond Fund Rating [Amount _|Duration [Mn USD_| in years AAA ‘Company Al 600] 7] ‘Company 8[ 300] 4] ‘Company C| 200] Za] AR ‘Company D| 400] Ei ‘Company | 350] a5} A Company F| 750] 1.5) [Total 72000] [Rating valuation matrix 14 years, 46.156 2.3 years, 7.5% 2.3 years, 7.7% 4.4 years, 15.4%% 4. An investment bank uses the Exponentially Weighted Moving Average (EWMA) technique with lambda of 0.9 to model the daily volatility of a security. The current estimate of the daily volatility is 1.5%. The closing price of the security is USD 20 yesterday and USD 18 today. Using continuously- compounded returns, what is the updated estimate of the volatility? a. 5.44% b. 3.62% c. 2:96% d. 1.31% Copyright © 2009 Global Association of Risk Professionals SO. Allrights reserved. ©OGARP nat asocaTioK oF nse PorEsiNNLS 2009 FRM Practice Exams 5. Consider two stocks A and B, Assume their annual returns are jointly normally distributed, the ‘marginal distribution of each stock has mean 2% and standard deviation 10%, and the correlation is (0.9. What is the expected annual return of stock A if the annual return of stock Bis 3%? a. 2.9% b. 2% & 11% d. 4.7% 6. In pricing a derivative using the Monte Carlo method, we need to simulate a reasonable number of paths for the price of the underlying asset. Suppose we use a simple model for the return of the underlying asset: vit) = drift*a, + vol * v A, * e(t), and e(t) is distributed ~ N(0,2), where drift and vol are known parameters and At is the step size, ‘The generation of each path requires a number of steps. Which of the following describes the correct procedure? a. Generate a random number from a normal distribution N(0,1), use the inverse normal function to get e(t), which will be fed into the model to get yit). Repeat the same procedure until you get the full desired path. b. Generate a random number from a normal distribution N(0,1), use the cumulative normal function to get e(t), which will be fed into the model to get y(t). Repeat the same procedure until you get the full desired path. © Generate a random number from a uniform distribution defined in [0,1], use the inverse ‘cumulative normal function to get e(t), which will be fed into the model to get y(t), Repeat the same procedure until you get the full desired path, 4d. Generate a random number from a uniform distribution defined in [0,1], use the cumulative normal function to get e(t), which will be fed into the model to get y(t). Repeat the same procedure until you get the full desired path. 7. Suppose that A and B are random variables, each follows a standard normal distribution, and the covariance between A and B is 0.35. What is the variance of (3A + 2B)? a 5.10 b. 14.47 © 9.20 d. 17.20 Copyright © 2009 Global Association of Risk Professionals. 51 All rights reserved. OG A R P 2009 FRM Practice Exams 8. You don’t have access to KMV's data. Your boss wants you to tell him your estimate of the probability of default of a credit. To do so, you use the Merton Model because the credit you are considering has no systematic risk. In Merton's Model, the distance to default (DD) and the expected default frequency (EDF) are a, positively and linearly related b. negatively and linearly related positively and nonlinearly related d._ negatively and non-linearly related 9, Suppose the rate on Company A's one-year zero-coupon bond is 10.0% and the one-year T-bill rate is 8.0%. Assume the T-bill is riskless and the probability of default of Company A’s bond is 10%. ‘What is the LGD of Company A’s bond? a. 18.18% b, 81.82% 20.01% d. 79.99%, 10. A bank is considering ways of significantly reducing or eliminating its credit exposure to defaults on a loan portfolio so that the bank's shareholders do not absorb the losses arising from such defaults. ignoring institutional issues (e.g., tax, accounting, capital requirements), three of the following programs have a similar impact on the credit risk of the bank. Which alternative fails to reduce credit risk? a, Sell the loan portfolio in its entirety to another bank. Borrow to finance an additional risk reserve to supplement existing loan-loss reserves. c. Securitize the loan portfolio. ¢. Buy credit protection on the loan portfolio with credit default swaps. 411, Consider a stock price S that follows a geometric Brownian motion dS = j: S dt + BS dz, with B strictly positive and 1 a fixed value. Which of the following statements is true? If the drift is negative, the price one year from now will be below today's price. The instantaneous rate of return on the stock follows a uniform distribution. The stock price $ follows a lognormal distribution. This model imposes mean reversion. aoge Copyright © 2009 Global Association of Risk Professionals $2 Allrights reserved. ©OGARP 2 23. 14. 15, ‘ani sour rms roressnats 2009 FRM Practice Exams ‘The joint probability distribution of random variables X and Y is given by f(x,y) = kxy for x= 4, 2, 3, y= 1, 2,3, and kis a positive constant. What is the probability that X + ¥ will exceed 5? v9 1/4 1/36 Cannot be determined Which of the following statements regarding Hypothesis Testing is incorrect? a. Hypothesis testing is used to make inferences about the parameters of a given population on the basis of statistics computed for a sample that is drawn from that population, b. Type Il error refers to the failure to reject the null hypothesis when its actually false. ©. The p-value decision rule is to reject the null hypothesis if the p-value is greater than the significance level. d. All else being equal, the decrease in the chance of making a Type | error comes at the cost of increasing the probability of making a Type ll error. If stock returns are independently identically normally distributed and the annual volatility is 30%, then the daily VaR at the 99% confidence level of a stock market portfolio is approximately: 2.41% 3.11% 4.40% 1.89% Asingle stock has a price of USD 10 and a current daily volatility of 2%. Using the delta-normal, ‘method, the VaR at the 95% confidence level of a long at-the-money call on this stock over a 1-day holding period is approximately: a. USD 0.23 b. USD 1.645, cc. USD0.33 d. usD0.16 Copyright © 2009 Global Association of Risk Professionals $3, All rights reserved. OG A R Ee 2009 FRM Practice Exams 16. A portfolio consists of two zero coupon bonds, each with a current value of USD 10. The first bond has a modified duration of 1 year and the second has a modified duration of 9 years. The yield curve is flat and all yields are 596. Assume all moves of the yield curve are parallel shifts. Given that the dally volatility of the yield is 1%, which of the following is the best estimate of the portfolio daily VaR at the 95% confidence level? a. USD2.33 b. USD 1.65 c. usD0.82 d. usp 1.16 17. Consider the following three methods of estimating the P&L of a bullet bond: full repricing, duration {PV01)}, and duration plus convexity. Ranking the estimated P&L impact of a large negative yield shock from the lowest P&L impact to the highest P&L impact, what is the ranking of the methods to estimate the P&L impact? duration plus convexity, duration, full repricing full repricing, duration plus convexity, duration duration, duration plus convexity, full repr duration, full repricing, duration plus convexity 18. Consider a position in a 5-year receive-fixed swap that makes annual payments on a USD 100 million notional. The floating leg has just been reset. The term structure is flat at 5%, the Macaulay duration of a 5-year par bond is 4.5 years, and the annual volatility of yield changes is 100bp. Your best estimate of the swap's VaR with 95% confidence over the next month is USD 1.6 mi USD 2.0 million USD5.5 million USD 7.1 million 19. Ifthe gold lease rate is higher than the risk-free rate, what is the market structure of the forward market for gold? a. Contango b. Backwardation Inversion d. Need more information to determine ‘Copyright © 2009 Global Association of Risk Professionals 54 Allrights reserved. ©OGARP ‘SLOPAL SOCIATION OF IS PROFESIONAL 2009 FRM Practice Exams 20. The price of a 3-year zero coupon government bond is 85.16. The price of a similar 4-year bond is 79.81. What is the one-year implied forward rate from year 3 to year 4? 5.4% 55% 5.8% 6.7% 21. A portfolio manager has a bond position worth USD 100 million. The position has a modified duration of 8 years and a convexity of 150 years. Assume that the term structure is flat. By how ‘much does the value of the position change if interest rates increase by 25 basis points? Usb -1,953,125 UsD -1,906,250 Usb -2,046,875 aoce 22. What is the annualized rate of return earned on a cash-and-carry trade entered into in March and closed out in June? 8.9% 9.8% 35.7% 39.1% ange 23. An investor sells a June 2008 call of ABC Limited with a strike price of USD 45 for USD 3 and buys a June 2008 call of ABC Limited with a strike price of USD 40 for USD 5. What is the name of this strategy and the maximum profit and loss the investor could incur? Bear Spread, Maximum Loss USD 2, Maximum Profit USD 3 Bull Spread, Maximum Loss Unlimited, Maximum Profit USD 3 Bear Spread, Maximum Loss USD 2, Maximum Profit Unlimited Bull Spread, Maximum Loss USD 2, Maximum Profit USD 3 aore Copyright © 2009 Global Association of Risk Professionals $5. Allrights reserved. ©OGARP ‘ugna aSSQCUATION OF mK PHoresSONAAS 2009 FRM Practice Exams 24. Which of the following problems are NOT inherent disadvantages of the historical simulation approach to estimating VaR? |. Itgives too little weight to more recent observations II. For long-only portfolios, itis likely to understate VaR following a recent structural increase in volatilities lil, ttalways ignores the fat tails present in the distribution of returns on many financial assets IV. Because of the delta approximation, it inadequately measures the risk of nonlinear instruments a. Land ttonly b. only ¢. Ijllland lV only d._illand Vv only 25. Abank holds USD 60 million worth of 10-year 6.5% coupon bonds that are trading at a clean price of 101.82. The bank is worried by the exposure due to these bonds but cannot unwind the position for fear of upsetting the client. Therefore, it purchases a total return swap (TRS) in which it receives annual Libor + 100 bps in return for the mark-to-market return on the bond. For the first year, the Libor sets at 6.25% and by the end of the year the clean price of the bonds is at 99.35, The net receipt/payment for the bank in the total return swap will be: Receive USD 2.23 million Receive USD 1.93 million Pay USD 1.93 million Pay USD 2.23 million Copyright © 2009 Global Association of Risk Professionals $6. All ights reserved. OGARP ‘SLaBA ASSOcATION OF sk PHorEsSoNALS 2009 FRM Practice Exams 26. Which of the following trade(s) contain basis risk? z. 28, Long 1,000 lots Nov 07 ICE Brent Oil contracts and short 1,000 lots Nov 07 NYMEX WTI Crude Oil contracts Long 1,000 lots Nov 07 ICE Brent Oil contracts and long 2,000 lots Nov 07 ICE Brent Oil at-the- money put Ill, Long 1,000 lots Nov 07 ICE Brent Oil contracts and short 1,000 lots Dec 07 ICE Brent Oil contracts 1V, Long 1,000 lots Nov 07 ICE Brent Oil contracts and short 1,000 lots Dec 07 NYMEX WTI Crude Oil contracts 1) W&IV Wi &Iv LV According to put-call parity, buying a put option on a stock is equivalent to: Buying a call option and buying the stock with funds borrowed at the risk-free rate. Selling a call option and buying the stack with funds borrowed at the risk-free rate. Buying a call option, selling the stock and investing the proceeds at the risk-free rate. Selling a call option, selling the stock and investing the proceeds at the risk-free rate. ‘A 3 month futures contract on an equity index is currently priced at USD 1000, the underlying index stocks are valued at USD 990 and pay dividends at a continuously-compounded rate of 2 percent and the current continuously compounded risk-free rate is 4 percent. The potential arbitrage profit er contract, given this set of data, is closest to UsD 10.00 usp 7.50, USD 5.00 USD 1.50 Copyright © 2009 Global Association of Risk Professionals 57 All rights reserved. 29, 30, a1, 2009 FRM Practice Exams Research and model projections indicate that a specific event is likely to move the CHF against the USD. While the direction of the move is highly uncertain, it is highly likely that magnitude of the move will be ant. Based on this information, which of the following strategies would provide the largest economic benefit? 2, Longa call option on USD/CHF and short a put option on USD/CHF with the same strike price and expiration date b, Longa call option on USD/CHF and long a put option on USD/CHF with the same strike price and expiration date c. Short a call option on USD/CHF and long a put option on USD/CHF with the same strike price and expiration date d, Short a call option on USD/CHF and short a put option on USD/CHF with the same strike price and expiration date Initially, the call option on Big Kahuna Ine. with 90-days to maturity trades at USD 1.40. The option has a delta of 0.5739. A dealer sells 200 call option contracts and to delta-hedge the position, the dealet purchases 11,478 shares of the stock at the current market price of USD 100 per share. The following day, the prices of both the stock and the call option increase, Consequently, delta increases to 0.7040. To maintain the delta hedge, the dealer should: Purchase 2,602 shares Sell 2,602 shares Purchase 1,493 shares Sell 1,493 shares Which of the following strategies creates a calendar spread? ‘a, Sell a call option with a certain strike price and buy a longer maturity call option with the same strike price, b. Buy a call option with a certain strike price and buy a longer maturity call option with the same strike price. Sella call option with a certain strike price and buy a shorter maturity call option with the same strike price. d._Buya call option with a certain strike price and sell a longer maturity call option with the same strike price. Copyright © 2009 Global Association of Risk Professionals 58. Allrights reserved. OGARP 32, 33, 34, 35. ona associat or mK nrestNALS 2009 FRM Practice Exams Which of the following underlying macro-economic conditions would leave an emerging market most vulnerable to the contagion effects of a currency crisis? Large current account surplus, low foreign exchange reserves, non-convertible currency Large current account deficit, low foreign exchange reserves, fully convertible currency Small current account deficit, high foreign exchange reserves, non-convertible currency Large current account surplus, high foreign exchange reserves, fully convertible currency aoge Consider an FRA (forward rate agreement) with the same maturity and compounding frequency as a Eurodollar futures contract. The FRA has a LIBOR underlying. Which of the following statements are true about the relationship between the forward rate and the futures rate? a. They should be exactly the same. b. The forward rate is normally higher than the futures rate, The forward rate is normally lower than the futures rate. d. They have no fixed relationship. Your bank is an active player in the commodity market. The view of the economist of the bank is that inflation is expected to rise moderately in the near term and market volatility is expected to remain low. The traders are advised to undertake deals on the metals exchange to align your book to conform with the expectations of the economist of the bank. As risk manager, you are asked to monitor the positions of the traders to make sure that they have the exposures to inflation and market volatility sought by the bank. Which trader has taken an appropriate position among the traders you are monitoring? tion and with strike price equal to a. Trader A bought a call and a put, both with 90-days to ex, the existing spot level. Trader B bought a put option with a down-and-in knock in feature. © Trader C bought a call option at the existing spot levels and sold a call at a higher strike price, both with 90-days to expiration. 4d. Trader D sold a call and bought a put at the existing levels, both with 90-days to expiration. Considering options generally (i., not only plain vanilla calls and puts), which of the following statements about vega is correct? ‘An option holder can never be vega negative. A deep in the money up and out call option has a negative vega, ‘A deep out of the money up and out call option has a negative vega ‘A deep out of the money digital option has a negative vega. ange Copyright © 2009 Global Association of Risk Professionals 59. All rights reserved. OGARP ‘GBA Asvcianon or it rroesseNS 2009 FRM Practice Exams 36, To hedge against future, unanticipated, and significant increases in borrowing rates, which of the following alternatives offers the greatest flexibility for the borrower? Fixed for floating swap Interest rate collar Interest rate floor Call swaption 37. Assuming other things constant, bonds of equal maturity will still have different DVO1 per USD 100 face value. Their DVO1 per USD 100 face value will be in the following sequence of highest value to. lowest value: Zero coupon bonds, par bonds, premium bonds Premium bonds, par bonds, zero coupon bonds Premium bonds, zero coupon bonds, par bonds Zero coupon bonds, premium bonds, par bonds aoe 38, The information ratio of the Sterole US Fund for 2006 against the S&P 500, its benchmark index, is 1. For the same time period, the fund's Sharpe ratio is 2, the fund has a tracking error of 7% against the S&P 500, and the standard deviation of fund returns is 5%. The risk-free rate in the US is 4%. Calculate the return for the S&P 500 during the time period. 11% a. b. 7% ‘ d. 14% 39. A fund manager recently received a report on the performance of his portfolio over the last year. ‘According to the report, the portfolio return is 9.3%, with a standard deviation of 13.5%, and a beta 0f 0.83, The risk-free rate is 3.2%, the semi-standard deviation (R,) of the portfolio is 8.4%, and the tracking error of the portfolio to the benchmark index is 2.8%. What is the difference between the value of the fund's Sortino ratio (computed relative to the risk-free rate) and its Sharpe ratio? 0.274 1727 0.653 0.378 ‘Copyright © 2009 Global Association of Risk Professionals 60, All ights reserved. 2009 FRM Practice Exams 40. Your firm has no prior derivatives trades with its counterparty Super Bank. Your boss wants you to evaluate some trades she is considering. in particular, she wants to know which of the following. trades will increase your firm's credit risk exposure to Super Bank: I buying a put option I. selling a put option ll buying a forward contract WV. selling a forward contract a. Landi only b. Mand Vonly ©. land V only d. | llland IV only 41. Consider the following one-period transition matrix: Initial Next Period State Period a B Default State A 95% | 5% 0% B 10% | 80%] 10% Default | 0% 0% 100% Ifa company is originally in State A, what is the probability that the company will have defaulted strictly before the fourth transition period from now? 0.875% 0.500% 1.375% 1.875% 42. As an approximation, itis true that Default swap spread = Return of a risky bond + Return of a risk-free bond Default swap spread = Return of a risky bond — Return of a risk-free bond Default swap spread = Return of a risky bond x Return of a risk-free bond Default swap spread = Return of a risky bond x (1~Return of a risk-free bond) pose Copyright © 2009 Global Association of Risk Professionals 61 All rights reserved. OG ARP 2009 FRM Practice Exams 43, 45, 46. in aCDO, the SPV is typically a. AAA-rated b. Acrated © BBB-rated d. Not rated . A trader whose risk you are monitoring tells you that he wants to benefit from a credit spread widening due to a recession. Which of the following would be good trades for his strategy? Go long risky bonds and short risk-free bonds at the beginning of the recession. Short risky bonds and go long risk-free bonds at the beginning of the recession. ©. Sell credit default swaps on bonds with a low credit quality and buy credit default swaps on bonds with a higher credit quality at the beginning of the recession. d._ Sell credit default swaps on bonds with low credit quality and go long low credit quality bonds. Bank B has a EUR 100 million loan portfolio and has set aside a reserve to cover the first EUR 20 million in default-related losses. If the bank wants to acquire protection for the remaining EUR 80 million in risk exposure, which of the following solutions would work and would expose the bank to. the least amount of counterparty risk? a. Buy credit protection in a senior subordinated CDS that covers EUR 80 million in losses above the first EUR 20 million. b. Buy credit insurance for losses up to EUR 80 million in excess of EUR 20 million on the loan portfolio. Issue a credit-linked note in which interest and principal may be withheld from investors to cover up to EUR 80 million in losses above the first EUR 20 million on the loan portfolio. 4. All three of the above choices work and expose the bank to the same amount of counterparty risk, Mr. Rosengvist, Asset Manager at BCD Bank, holds a portfolio of SEK 200 million. The portfolio Consists of BBB-rated bonds. Assume that the one-year probability of default is 4%, the recovery ‘ate is 60%, and defaults are uncorrelated over years. What is the 2-year cumulative expected credit loss on Mr. Rosengvist's portfolio? a. SEK 6.35 million b. SEK 6.40 million cc. SEK 9.48 million d. SEK 9.60 million Copyright © 2009 Global Association of Risk Professionals 62 llrightsreserved. OGARP ston assoion OF st wRTessons 2009 FRM Practice Exams 47, Using the Merton model, the value of the debt increases if all other parameters are fixed and I The value of the firm decreases ll, The riskless interest rate decreases Time to maturity increases IV. The volatility of the firm value decreases a. land tony b, land IV only c. Mandi only d. Mand iV only is going to buy 10,000 barrels of West Texas Crude Oil. It plans to hedge the purchase using the Brent Crude futures contract. The correlation between the spot and futures prices is 0.72. The volatility of the spot price is 0.35 per year. The volatility of the Brent Crude futures price is 0.27 per year. What is the hedge ratio for the firm? 0.5554 0.9333 1.2099 0.8198 eore 49, Itis June 2" and a fund manager with USD 10 million invested in government bonds is concerned that interest rates will be highly volatile over the next three months. The manager decides to use the September Treasury bond futures contract to hedge the value of the portfolio. The current futures price is 95.0625. Each contract is for the delivery of USD 100,000 face value of bonds. The duration of the manager's bond portfolio in three months will be 7.8 years. The cheapest to deliver bond in the Treasury bond futures contract is expected to have a duration of 8.4 years at maturity of the contract, At the maturity of the Treasury bond futures contract, the duration of the underlying benchmark Treasury bond is 9 years. What position should the fund manager undertake to mitigate his interest rate risk exposure? a. Short94 contracts b. Short98 contracts . Short105 contracts d. Short113 contracts Copyright © 2009 Global Association of Risk Professionals 63, All rights reserved. ©OGARP soa ssocunon oF st rerssoMs 2009 FRM Practice Exams 50. A bond trader has bought a position in Treasury Bonds with a 4% annual coupon rate on February 15, 2015, The DVO1 of the position is USD 80,000. The trader decides to hedge his interest rate risk with the 4.5% coupon rate Treasury Bonds maturing on May 15, 2017 which has a DVO1 of 076 per USD 100 face value. To implement this hedge, approximately what face amount of the 4.5%, Treasury bonds maturing on May 15, 2017 should the trader sell? Usb 80,000 UsD 10,500,000 USD 80,000,000 USD 105,000,000 ares END OF 2009 FRM FULL EXAM PRACTICE EXAM I Copyright © 2009 Global Association of Risk Professionals 64 All rights reserved. 2 £ S = a y un t £ a = m4 i a 3 3 a 2009 FRM Full Exam Practice Exam | Answer Key 00000900000000009090000008 All ights reserved. ©OGARP mM Assaunox ovis ures 2009 FRM Practice Exams This page is intentionally left blank. Copyright © 2009 Global Association of Risk Professionals 66 All rights reserved. ©OGARP SLL ASSOCIATION OF RSE POFESINALS 2009 FRM Practice Exams 2009 FRM Full Exam Practice Exam | Answers & Explanations 1. Given the information provided in the table below, what is the portfolio VaR, at the 99% confidence level, of the following 100 million CHF equally weighted investment portfolio? Expected = ° Asset ates Volatility | Correlation ‘Stocks Bonds Stocks| 24.00% 18% 1 Bonds | 15.00% 6% or 1 a. 27.96 million CHE b, 22.77 million CHE © 20.97 million CHF d. 13.98 million CHF CORRECT: B The variance of the equally weighted portfolio is 0.5"2 * 0.1842 + 0.542 * 0.06%2+2*0.5*0.5*0.1 * 0.18 * 0.06 = 0.081 + 0.0009 + 0.0005 = 0.00954. The volatility is then 9.77%. The portfolio VaR or the risk budget is 2.33 * 9.77% * 100 million CHF = 22.77 million CHF. Reference: Allen et al. Chapters 2,3. 2. You are asked by your boss to estimate the exposure of a hedge fund to the S&P 500. Though the fund claims to mark to market weekly, it does not do so and marks to market once a month. The fund also does not tell investors that it simply holds an ETF which is indexed to the S&P50O, Because of the claims of the hedge fund, you decide to estimate the market exposure by regressing weekly returns of the fund on the weekly return of the S&P500. Which of the following properties correctly describes a property of your regression estimates? a, The beta of your regression will be one because the fund holds the S&P S00. b. The beta of your regression will be zero because the fund returns are not synchronous with the S&P 500 returns. ©. The intercept of your regression will be positive, showing that the fund has a positive alpha When estimated using an OLS regression. dd. The beta will be misestimated because hedge fund exposures are non-linear. Copyright © 2009 Global Association of Rsk Professionals. 67 All rights reserved. OGARP CORRECT: C The alpha is spurious and results from the fact that returns are non-synchronous. d. is incorrect because the true exposure is linear. The beta is greater than zero and less than one because of non- synchroneity. Reference: Amenc and Le Sourd, Portfolio Theory and Performance Analysis. Chapter 4 3. The following table shows the composition of the GARP Bond Fund. What are the portfolio duration and portfolio yield of the fund? 2009 FRM Practice Exams [GARP Bond Fund [Rating [Amount [Duration [Nin USD_| in years [AAA ‘Company Al 600] 7 ‘Company 8] 300] 4| ‘Company C200] Z| AR ‘Company D] 400) 4] ‘Company E 350] os] os ‘Company 755] 73] [Total [Rating valuation matn Years 34 Rating AAA 625%| 6.75% 7.35%) 6.00%| BA 6.75%] _7.35%| _8.05%| _8.80%| A T75%| 8.45%] 9.15%] __9.85%| 14 years, 46.1% 2.3 years, 7.5% 2.3 years, 7.7% 44 years, 15.4%% The calculation of portfolio duration and portfolio yield is based on the proportional weightage of respective company to its duration and yield. The portfolio duration and portfolio yield after mapping the yield from rating matrix is as follows; CORRECT: 8 Copyright © 2009 Global Association of Risk Professionals Allrights reserved. OGARP Loma ssocuren oF mst pressions 2009 FRM Practice Exams This answer reflects the proportion of amount taken as weights to calculate the portfolio duration ‘and portfolio yield. [ARP Bond Fund [Amount [Proportion % [Duration _|Yield AAR COAL 600] 5] 675% CoB 300] 15% ‘4[_8.00%4 Co} 200] 10% 25] 7.35% AA, Copl 400] 20% a] 8.80% Coe| 350] 18% os] 6.75% lm CoF| 1) 8% T5]__ 6.45%] [rotal_— [2000] 100%] 2.30] 7.54% INCORRECT: A ~ If the candidate does a simple addition of the duration and the mapped yield, he would get this answer. INCORRECT: C— Though the portfolio duration is correct, but it is arrived by taking simple average of duration. However, if the candidate would take simple average of mapped yield instead of proportion his answer would be 7.7% and not 7.5% INCORRECT: D — if the candidate averages based on rating classes (3 AAA/AA/A) instead of the companies, he would get this answer. Reference: Tuckman; Chapter 6. 4, An investment bank uses the Exponentially Weighted Moving Average (EWMA) technique with lambda of 0.9 to model the daily volatility of a security. The current estimate of the daily volatility is 1.5%. The closing price of the security is USD 20 yesterday and USD 18 today. Using continuously- compounded returns, what is the updated estimate of the volatility? a. 5.44% b. 3.62% c. 2.96% d. 131% CORRECT: B The current return of the security is = In (18/20) = -10.536%. Using an EWMA model, the updated volatility is given as: V(t) = {lambda* ((V[t-1]*2) +(4 ~ lambda)*(current return’2)} 40.5 = (0.9 * ((0.01542) + (1 -0.9) * (-0.10536%2 }} 40.5 ‘Copyright © 2009 Global Association of Risk Professionals 69, Allrights reserved. ©OGARP GONE Assozuron oF is rtressionas 2009 FRM Practice Exams = 3.62% INCORRECT: A ~ Forgets to square the volatility terms INCORRECT: C Forgets to square the volatility terms and to take the square root of the resulting variance, then miscalculates conversion to percentage. INCORRECT: D - Forgets to take the square root of the variance, then miscalculates conversion to percentage. Reference : Hull, Chapter 21. 5. Consider two stocks A and B, Assume their annual returns are jointly normally distributed, the ‘marginal distribution of each stock has mean 2% and standard deviation 10%, and the correlation is 0.9. What is the expected annual return of stock A if the annual return of stock B is 3%? a 2.9% b. 2% c 11% d. 4.7% CORRECT: A Ele Ifo =4) = Ha + (Pea vor/ Pllx— 44) = 0.02 + 0.9 * (0,03 ~0.02) = 0.029 Reference: Damodar Gujarati, chapter 2. 6. In pricing a derivative using the Monte Carlo method, we need to simulate a reasonable number of paths for the price of the underlying asset. Suppose we use a simple model for the return of the underlying asset: y(t) = drift*4, + vol * Vv 4, * e(t), and e(t) is distributed ~ N(O,1), where drift and vol are known parameters and 4, is the step size ‘The generation of each path requires a number of steps. Which of the following describes the correct procedure? @, Generate a random number from a normal distribution N(0,1), use the inverse normal function to get e(t), which will be fed into the model to get y(t). Repeat the same procedure until you get the full desired path. Copyright © 2009 Global Association of Risk Professionals 70 Allrights reserved. OG A R P 2009 FRM Practice Exams b. Generate a random number from a normal distribution N(0,1), use the cumulative normal function to get e(t), which will be fed into the model to get yit), Repeat the same procedure until you get the full desired path. ©. Generate a random number from a uniform distribution defined in [0,1], use the inverse ‘cumulative normal function to get e(t), which will be fed into the model to get y(t). Repeat the same procedure until you get the full desired path. d, Generate a random number from a uniform distribution defined in [0,1], use the cumulative normal function to get e(t), which will be fed into the model to get y(t). Repeat the same procedure until you get the full desired path, CORRECT: C This question wants to test if the candidate knows the basic steps to generate a very simple path: ‘answering this question means that the candidate would be able to build a simple spreadsheet showing the Monte Carlo logic. The correct procedure is the one described in c); while a), b) and d) are nonsensical calculations. Reference: Philippe Jorion, Value at Risk, The New Benchmark for Managing Financial Risk, 3” edition (New York: McGraw-Hill, 2007), Chapter 12, 7. Suppose that A and B are random variables, each follows a standard normal distribution, and the covariance between A and B is 0.35, What is the variance of (3A +28)? a. 15.10 b. 14.47 c. 9.20 d. 17.20 CORRECT: D Since each variable is standardized, its variance is 1. Therefore V(3A+2B) = 9 V(A) +4 V(B) + 2x3 x2 xCov(A,B) = 944442 = 17.2 INCORRECT: A~ 9+4+6*0,35=15.1 INCORRECT: B— 944 +12 *0,3592= INCORRECT: C— 3+2+12*0,35=9.2 Reference: Damodar Gujarati Copyright © 2009 Global Association of Risk Professionals 71 All ights reserved. ©OGARP om asecunn ok orsNNS 2009 FRM Practice Exams 8. You don’t have access to KMV's data. Your boss wants you to tell him your estimate of the probability of default of a credit. To do so, you use the Merton Model because the credit you are lering has no systematic risk. In Merton's Model, the distance to default (DD) and the expected default frequency (EDF) are con: positively and linearly related negatively and linearly related positively and nonlinearly related negatively and non-linearly related CORRECT: D The risk neutral probability of default, EDF, in the Merton Model is 1- N(d,). The higher the distance to default, DD DD =d,= . the lower the risk neutral probability of default is. On the contrary, the lower DD, the higher EDF is. The relationship is non-linear. When the DD is low, EDF is high. If DD is imminent, EDF is high as well. Similarly, if DD Is high, EDF is small and not imminent Reference: De Servigny and Renault, Measuring and Managing Credit Risk, Chapter 3. 9. Suppose the rate on Company A’s one-year zero-coupon bond is 10.0% and the one-year T-bill rate is 8.0%. Assume the T-bill is riskless and the probability of default of Company A’s bond is 10%. What is the LGD of Company A’s bond? 18.18% 81.82% 20.01% 79.99% CORRECT: A (1+10%)*(1-PO)+(14105)*PD*(1-LGD)=1+8% 1.10.94 1.1 x0.10 x (1~LGD) = 1.08 0.99 + 0.11 x (1 -LG0, 0.11 x((1 - LGD) = 1.08 -0.99 (1-LGD) = (1.08 -0.99) 0,11 LGD = 1- (1.08 0.99) /0,11 = 18.18% Copyright © 2009 Global Association of Risk Professionals 72 All rights reserved. 2009 FRM Practice Exams LGD = 1 - (141) ~ (141) x (1 — PO))/{(1+r) x PO) Reference: De Servigny and Renault, Measuring and Managing Credit Risk, Chapter 3, 4. 10. A bank is considering ways of significantly reducing or eliminating its credit exposure to defaults on a loan portfolio so that the bank’s shareholders do not absorb the losses arising from such defaults, \gnoring institutional issues (e.g., tax, accounting, capital requirements), three of the following Programs have a similar impact on the credit risk of the bank. Which alternative fails to reduce credit risk? Sell the loan portfolio in its entirety to another bank. Borrow to finance an additional risk reserve to supplement existing loan-loss reserves. Securitize the loan portfoli Buy credit protection on the loan portfolio with credit default swaps. aoge CORRECT: B All three of the other choices are economically equivalent. Selling loans to an external party eliminates all credit risk for the institution. Similarly, securitizing the loan portfolio removes the loans from the bank’s books and eliminates the credit risk for the institution. Buying credit protection using credit default swaps will offer protection against credit risk. This alternative implies counterparty risk, Borrowing does not work in the long run because shareholders still at some point have to take the hit for default-related losses. Additionally, the increased borrowing to finance the loan loss reserves will increase the risk for the shareholders. Reference: Culp, Chapter 16 11. Consider a stock price S that follows a geometric Brownian motion dS = yS dt + BS dz, with B strictly Positive and j.a fixed value. Which of the following statements is true? a. Ifthe drift jis negative, the price one year from now will be below today's price, b, The instantaneous rate of return on the stock follows a uniform distribution, The stock price $ follows a lognormal distribution. 4d. This model imposes mean reversion. CORRECT: C Copyright © 2009 Global Association of Risk Professionals 73 All rights reserved. OGARP UAL sco oF FrEsNAS 2009 FRM Practice Exams INCORRECT: A ~The expected price is less than today’s price, but not the price in all the states of world. INCORRECT: B— The instantaneous rate of return on the stock follows normal distribution, INCORRECT: D = This model does not impose mean reversion. i Reference: Philippe Jorion, Value at Risk: The New Benchmark for Managing Financial Risk, 3” ed. (New York: McGraw-Hill, 2007). Chapter 12 12. The joint probability distribution of random variables X and ¥ is given by f(x,y) = kxy for x= 1, 2, 3, y= 1, 2, 3, and kis a positive constant, What is the probability that X + will exceed 5? 1/9 4 1/36 Cannot be determined aooe CORRECT: B Note that yy fny)= sa pt Substituting the various values of x and y, we get f(1,1)=k, f(1,2)=2k, f(1,3)=3k, fl(2,1)=2k, 2,2)=4k, ‘f{2,3)6k, f(3,1)=3k, f(3,2)=6k, and f(3,3)=9k. Therefore, ki + 2k + 3k + 2k + 4k + 6k + 3k + 6K + 9) so that, 36k = 1 and k=1/36. P(X#Y>5) = (3,3) = 1/363 x3 = 1/4 Reference: Damodar Gujarati 13. Which of the following statements regarding Hypothesis Testing is incorrect? ‘a. Hypothesis testing is used to make inferences about the parameters of a given population on the basis of statistics computed for a sample that is drawn from that population. Type Il error refers to the failure to reject the null hypothesis when it is actually false. ©. The p-value decision rule is to reject the null hypothesis if the p-value is greater than the significance level. 4. Allelse being equal, the decrease in the chance of making a Type | error comes at the cost of increasing the probability of making a Type I! error. Copyright © 2009 Global Association of Risk Professionals 74 All rights reserved. 2009 FRM Practice Exams CORRECT: C The true statement is to reject Ho if the p-value is smaller than the significance level. INCORRECT: A ~ regarding the primary use of Hypothesis Testing. INCORRECT: B - regarding the definition of type I error. INCORRECT: D ~ type | error and type I! error are in tradeoff. Reference: Damodar Gujarati 14, If stock returns are independently identically normally distributed and the annual volatility is 30%, then the daily VaR at the 99% confidence level of a stock market portfolio is approximately: 2.41% 3.11% 4.40% 1.89% CORRECT: C The 1-day volatility is s * (1/252)*0.5 = 0.3 * 0.629941 = 0.018898, The VaR at the 99% confidence level is then equal to 2.32635 * 0.018898 = 4.40% INCORRECT: A ~One gets A if one uses 1.645 instead of 2.326, INCORRECT: B— One gets B if one uses the monthly volatility instead of the daily one INCORRECT: D~ One gets D is the daily volatility. Reference: Allen, Boudoukh and Saunders, 2004, chapter 1, p 6-8 15. A single stock has a price of USD 10 and a current daily volatility of 2%. Using the delta-normal method, the VaR at the 95% confidence level of a long at-the-money call on this stock over a 1-day holding period is approximately: usp 0.23, USD 1.645 usp 0.33 USD 0.16 aooe CORRECT: D Copyright © 2009 Global Association of Risk Professionals 75, Allrights reserved. OG A R P 2009 FRM Practice Exams This question requires candidates to know the formula for the delto-normal VaR approximation, and also to know that the delta of an at-the-money call is 0.5. VaR =| A |x1.645 x ox S = 0.5% 1,645 x 0,02 x10 = 0.1645 INCORRECT: A~ We get A by using 2.326 instead of 1.645 INCORRECT: B— We get B if we use 2 instead of 2% for the volatility INCORRECT: C— We get Cif we use a delta of 1 Reference: Allen et al, Chapter 3 16. A portfolio consists of two zero coupon bonds, each with a current value of USD 10, The first bond has a modified duration of 1 year and the second has a modified duration of 9 years. The yield curve is flat and all yields are 5%. Assume all moves of the yield curve are parallel shifts. Given that the daily volatility of the yield is 1%, which of the following is the best estimate of the portfolio daily VaR at the 95% confidence level? Usp 2.33, USD 1.65. USD 0.82. USD 1.16 aege CORRECT: B This question assesses candidates’ abilities to apply the duration VaR formula to two bonds simultaneously and to recall that the duration of a zero coupon bond is equal to the bond maturity. Using an obvious extension of Jorion’s equation 9.5, VaR = D, xV, x1.645x 0 + D, XV, x1.645x0 = (D, xV, + D, xV,)x1.645% 0 = (D, + D,)*10x1.645x0 = 10%101.645% 0.01 = 1.645 INCORRECT: A ~ Is the 99% confidence level VaR INCORRECT: C—Arises if the candidate mistakenly divides the correct answer by the number of bonds INCORRECT: D ~ Makes both mistakes Reference: Tuckman Copyright © 2009 Global Association of Risk Professionals 76: All rights reserved, OGARP ona sso TON Fw rE 2009 FRM Practice Exams 17. Consider the following three methods of estimating the P&L of a bullet bond: full repricing, duration {PVO1), and duration plus convexity. Ranking the estimated P&L impact of a large negative yield shock from the lowest P&L impact to the highest P&L impact, what is the ranking of the methods to estimate the P&L impact? a. duration plus convexity, duration, full repricing full repricing, duration plus convexity, duration ©. duration, duration plus convexity, full repricing d. duration, full repricing, duration plus convexity CORRECT: C The price / yield line with yield on the x axis and price on the y axis Is convex to the origin. The duration at any yield level is the tangent to that curve. Therefore, except at the exact point of tangency, duration will always underestimate the price change. INCORRECT: A ~ Duration will always underestimate price change for negative yield shocks INCORRECT: Full repricing will never generate a smaller positive price change than duration because duration represents the point of tangency INCORRECT: D — Full repricing will generate a higher price for a large negative yield change than wil duration plus convexity Reference: Allen, Boudoukh, Saunders, Chapter 3 18. Consider a position in a 5-year receive-fixed swap that makes annual payments on a USD 100 million notional. The floating leg has just been reset. The term structure is flat at 5%, the Macaulay duration of a 5-year par bond is 4.5 years, and the annual volatility of yield changes is 100bp. Your best estimate of the swap’s VaR with 95% confidence over the next month is a, USDL6 million b, USD 2.0 million USD5.5 million d, USD 7.1 million CORRECT: A Because the floating-rate leg has just been reset, its duration is 1. Net duration is 4.5-1=3.5 year, or modified duration of 3.5/1.05=3.33. The 95% VaR of monthly changes in yields is 1.65°1%/ 12 = 0.48%. Multiplying, this gives USD 100*0.48%*3.33=USD 1.588 Copyright © 2009 Global Association of Risk Professionals 77 All rights reserved. ©OGARP LON ASHOCUTION OF RK PROFESSIONS 2009 FRM Practice Exams INCORRECT: B ~ This uses a net duration of 4.5 years and ignores the duration of the floating-rate leg. INCORRECT: C~ This is the annual VaR, but should be translated to a monthly horizon. INCORRECT: D ~ This is the annual VaR computed by ignoring the duration of the floating-rate leg. Reference: Hull, Chapter, Chapter 7 19. If the gold lease rate is higher than the risk-free rate, what is the market structure of the forward market for gold? a. Contango b. Backwardation Inversion d. Need more information to determine CORRECT: B Alease rate higher than the risk fee rate will force a negatively sloped forward curve, Le. backwardation INCORRECT: A The forward price = spot*exp( risk free rate - lease rate). If the lease rate is higher than the risk free rate, forwards will be lower than spot, implying contango INCORRECT: C— The term inversion is used to describe yield curves, not commodity forwards INCORRECT: D ~ There is enough information in the question to provide an answer Reference: MacDonald, Chapter 6 20. The price of a 3-year zero coupon government bond is 85.16. The price of a similar 4-year bond is 79.81, What is the one-year implied forward rate from year 3 to year 4? 5.4% 5.5% 5.8% 6.7% ange ‘CORRECT: D Copyright © 2009 Gio Allrights reserved. | Association of Risk Professionals 78 2009 FRM Practice Exams Price of three bond _ 85.16 rice of four yearbond 79.81 Forward rate = 0.087034 or 6.7% INCORRECT: A ~ Is a combination of 8 and C ~Is the return of the 3-year bond Is the return of the 4-year bond Reference: Hull, Tuckman 21. A portfolio manager has a bond position worth USD 100 million. The position has a modified duration of 8 years and a convexity of 150 years. Assume that the term structure is flat. By how ‘much does the value of the position change if interest rates increase by 25 basis points? USD 1,953,125 USD -1,906,250 Usp -2,046,875 Uso -2,187,500 CORRECT: A AV =-Dyoa x Ay V+ 0.5% Convexity Ay? xV AV =-8% 0.0025 100M +0.5%150-x (0.0025)? x100M, AV=-2M +46,875 AV =-1,953,125 INCORRECT: B ~ Omits 0.5 from the second term INCORRECT: C- Subtracts the second term INCORRECT: D - Makes both mistakes Reference: Tuckman 22. What is the annualized rate of return earned on a cash-and-carry trade entered into in March and closed out in June? 8.9% 9.8% 35.7% 39.1% pore Copyright © 2009 Global Association of Risk Professionals 79. Allrights reserved. OG A R E 2009 FRM Practice Exams CORRECT: C By formula Far = See” + C, where Far= June forward price, Sp = March forward price, r= risk free interest rate, T=/length of cash-and-carry, C= storage cost Solving 5.90 = 5.35e"""? + 0.05 Solution is = 35.7% INCORRECT: A ~8,9 = LN((5.9-0.05)/5.35) (forgets to annualize the return) INCORRECT: B ~ 9.8 = LN((5.9)/5.35) (forgets to include the storage cost and to annualize the return) INCORRECT: D~ 39.1 = (12/3)LN((5.9)/5.35) — 0.05 (forgets to include the storage cost) Reference: Robert L McDonald, Derivatives Markets, Chapter 6 23. An investor sells a June 2008 call of ABC Limited with a strike price of USD 45 for USD 3 and buys a June 2008 call of ABC Limited with a strike price of USD 40 for USD 5. What is the name of this strategy and the maximum profit and loss the investor could incur? Bear Spread, Maximum Loss USD 2, Maximum Profit USD 3 Bull Spread, Maximum Loss Unlimited, Maximum Profit USD 3 Bear Spread, Maximum Loss USD 2, Maximum Profit Unlimited Bull Spread, Maximum Loss USD 2, Maximum Profit USD 3 apeo CORRECT: D Buying a call option at lower stock price and selling call option at higher strike price is called as Bull Spread. Bear Spread is buying the call option at higher price and selling the call at lower strike price. The Cost of strategy will be USD 3-USD 5 = -USD 2 The Payoff, when Stack price S+< USO 40 will be -USD 2 (the cost of strategy) as none of the option will be exercised, The Payoff, when stock price 5-245, (as both options will be exercise) will be USD 5, Since the cost of strategy is -USD 3, hence profit will be USD 5-USD 2 = USD 3 When Stock price is USD 40< S;> USD 45, Only the call option bought by the Investor would be exercised hence the pay off will be S;~40, since the cost of strategy is -USD 3, The Net profit will be 51-43, which would always be lower than USD 3. Reference: Hull, Chapter 10- Trading Strategies involving Options. Copyright © 2009 Global Association of Risk Professionals 80, All ights reserved. OG A R P 2009 FRM Practice Exams 24, Which of the following problems are NOT inherent disadvantages of the historical simulation approach to estimating VaR? |. It gives too little weight to more recent observations |. For long-only portfolios, itis likely to understate VaR following a recent structural increase in volatilities Ii, Iealways ignores the fat tails present in the distribution of returns on many financial assets IV. Because of the delta approximation, it inadequately measures the risk of nonlinear instruments a. land ttonly b. Wonly c, |, Mand IV only 4, Mland IV only CORRECT: C The disadvantage with the Historical Simulation Model is that it may not recognize the changes in volatility and correlation following recent structural changes. The model can be adjusted so that it I & IV, are disadvantages of Monte gives more weight to recent observations. The other options, Carlo method and Delta-normal method. Reference: Allen et al. Chapters 2,3, 25. Abank holds USD 60 million worth of 10-year 6.5% coupon bonds that are trading at a clean price of 1101.82. The bank is worried by the exposure due to these bonds but cannot unwind the position for fear of upsetting the client. Therefore, it purchases a total return swap (TRS) in which it receives annual Libor + 100 bps in return for the mark-to-market return on the bond. For the first year, the Libor sets at 6.25% and by the end of the year the clean price of the bonds is at 99.35. The net receipt/payment for the bank in the total return swap will be: Receive USD 2.23 million, Receive USD 1.93 million. Pay USD 1.93 million. Pay USD 2.23 million. CORRECT: Copyright © 2009 Global Association of Risk Professionals 81 All rights reserved, 2009 FRM Practice Exams it’s the result of this calculation: the notional amount is 60 million USD . Therefore the bank will receive the interest payment linked to the LIBOR rate: 60 million USD * (6,25%+100 bp) = 4. 35 million USD. The bank will pay the fixed coupon plus the change in the value of the bond: 60 million USD * 6.5% + 60 million *(99.35%-101,82%) = 2.418 million USD. Hence the total net amount the bank will receive is: 4.35 million USD - 2.418 million USD = 1.932 mitfion USD Reference: Hull Chapter 7 - Swops 26, Which of the following trade(s) contain basis risk? Long 1,000 lots Nov 07 ICE Brent Oil contracts and short 1,000 lots Nov 07 NYMEX WTI Crude Oil contracts Ml, Long 1,000 lots Nov 07 ICE Brent Oil contracts and long 2,000 lots Nov 07 ICE Brent Oil at- the-money put I. Long 1,000 lots Nov 07 ICE Brent Oil contracts and short 1,000 lots Dec 07 ICE Brent Oil contracts IV. Long 1,000 lots Nov 07 ICE Brent Oil contracts and short 1,000 lots Dec 07 NYMEX WTI Crude Oil contracts a 1&ill b. N&IV cc Ml&iV dma CORRECT: D Basis Risk is spread risk, which arise from trading the spread (long and short 2 positively correlated assets or some asset with different expiration) | is spread trade in highly correlated asset with same expiration month I faces with gamma and vega risk is spread trade in trading the flattening of the forward curve Vis spread trade in trading 2 assets with different expiration date Reference: Robert L, McDonald, Derivatives Markets (Boston: Addison-Wesley, 2003), Chapter 6. Copyright © 2009 Global Association of Risk Professionals 82 All rights reserved. 2009 FRM Practice Exams 27. According to put-call parity, buying a put option on a stack is equivalent to: Buying a call option and buying the stock with funds borrowed at the risk-free rate. Selling a call option and buying the stock with funds borrowed at the risk-free rate. Buying a call option, selling the stock and investing the proceeds at the risk-free rate. Selling a call option, selling the stock and investing the proceeds at the risk-free rate CORRECT: C Buying a call option, selling the stock and investing the proceeds at the risk-free rote. INCORRECT: A — Buying a call option is correct, but the rest of the statement is incorrect. INCORRECT: 8 ~The entire statement is incorrect. INCORRECT: D— Selling a call option is incorrect, but the rest of the statement is correct. Reference: Options, Futures, and Other Derivatives, 6” edition, by John Hull, Chapter 10. 28. A3 month futures contract on an equity index is currently priced at USD 1000, the underlying index stocks are valued at USD 990 and pay dividends at a continuously-compounded rate of 2 percent and the current continuously compounded risk-free rate is 4 percent. The potential arbitrage profit per contract, given this set of data, is closest to a. USD 10.00 b. usD7.50 . usD5.00 d. USD1,50 CORRECT: C According to the fundamental pricing relationship between spot assets and the associated futures, the futures price, to prevent arbitrage, should equal 990 x e (0.04 ~0.02) x 0.25 or 995. Hence, the futures contract is overvalued, indicating it should be sold and the index should be purchased for an arbitrage profit of USD 1000 - USD 995 = USD 5 Reference: Hull, Options, Futures, and Other Derivatives, 6” ed. Chapter 5 Copyright © 2009 Global Association of Risk Professionals $3, Allrights reserved. OG A R P 2009 FRM Practice Exams 29, Research and model projections indicate that a specific event is likely to move the CHF against the USD. While the direction of the move is highly uncertain, it is highly likely that magnitude of the ‘move will be significant. Based on this information, which of the following strategies would provide the largest economic benefit? a. Longa call option on USD/CHF and short a put option on USD/CHF with the same strike price and expiration date b. Longa call option on USD/CHF and long a put option on USD/CHF with the same strike price and expiration date ©. Short a call option on USD/CHF and long a put option on USD/CHF with the same strike price and expiration date d. Short a call option on USD/CHF and short a put option on USD/CHF with the same strike price and expiration date CORRECT: The question tests on understanding of a “straddle” strategy and its application on currency trading. Along straddle strategy involves buying (long) a call and put option with the same strike price and expiration date, and will benefit most when the underlying moves away from the current equlibrium. Long call and long put create a straddle. INCORRECT: A ~ It sells a put option while it should buy one put. . INCORRECT: C—It sells a call option while it should buy one call. INCORRECT: D — It sells both the call and put option while it should buy bath. Reference: Hull. 30. initially, the call option on Big Kahuna Inc. with 90-days to maturity trades at USD 1.40. The option has a delta of 0.5739. A dealer sells 200 call option contracts and to delta-hedge the position, the dealer purchases 11,478 shares of the stock at the current market price of USD 100 per share. The following day, the prices of both the stock and the call option increase. Consequently, delta increases to 0.7040, To maintain the delta hedge, the dealer should: a. Purchase 2,602 shares. b. Sell 2,602 shares. . Purchase 1,493 shares. d, Sell 1,493 shares. CORRECT: A ‘Number of calls = 200 contracts x 100 = 20,000 calls. Copyright © 2009 Global Association of Risk Professionals 84 All rights reserved. OG A R P 2009 FRM Practice Exams Number of shares = (Number of calls) x (New delta ~ Old delta) = 20,000 x (0.7040 - 0.5739) = #2,602 shares Positive sign indicates that the manager should purchase new shares. INCORRECT: B ~ Because the formula is incorrect, ie. old delta minus new delta. INCORRECT: C~ Because the number of shares (instead of number of calls) is used in the calculation. INCORRECT: D ~As per explanation in ‘C’ above and sign error. Reference: Hull. 311. Which of the following strategies creates a calendar spread? a. Sell_2 call option with a certain strike price and buy a longer maturity call option with the same strike price. 'b. Buya call option with a certain strike price and buy a longer maturity call option with the same strike price. Sell a call option with a certain strike price and buy a shorter maturity call option with the same strike price. d. Buya call option with a certain strike price and sell a longer maturity call option with the same strike price. CORRECT: A INCORRECT: B~ As buy a call option, INCORRECT: C—As buy a shorter-maturity call option INCORRECT: D—As this is a reverse calendar spread. Reference: Hull 32. Which of the following underlying macro-economic conditions would leave an emerging market ‘most vulnerable to the contagion effects of a currency crisis? a. Large current account surplus, low foreign exchange reserves, non-convertible currency b. Large current account deficit, low foreign exchange reserves, fully convertible currency €. Small current account deficit, high foreign exchange reserves, non-convertible currency d. Large current account surplus, high foreign exchange reserves, fully convertible currency CORRECT: B Copyright © 2009 Global Association of Risk Professionals 85, All rights reserved. ©OGARP con assoruron oF Rs reeressON 2009 FRM Practice Exams INCORRECT: A ~ Large current account surplus and non-convertible currency would protect the local currency INCORRECT: C— High foreign exchange reserves and non-convertible currency would protect the local currency INCORRECT: D ~ Large current account surplus and high foreign exchange would protect the local currency Reference: Saunders, Chapter 15, Foreign Exchange Risk 33. Consider an FRA (forward rate agreement) with the same maturity and compounding frequency as a Eurodollar futures contract. The FRA has a LIOR underlying. Which of the following statements are true about the relationship between the forward rate and the futures rate? a, They should be exactly the same. b, The forward rate is normally higher than the futures rate. ©. The forward rate is normally lower than the futures rate. d, They have no fixed relationship, CORRECT: C As Eurodollar futures contract is marked to market and settled daily, normally forward rate is adjusted lower, so called convexity adjustment, by: Forward rate = Futures rate Lahh, 2 Reference: Hull 34. Your bank is an active player in the commadity market. The view of the economist of the bank is that inflation is expected to rise moderately in the near term and market volatility is expected to remain low, The traders are advised to undertake deals on the metals exchange to align your book to conform with the expectations of the economist of the bank. As risk manager, you are asked to ‘monitor the positions of the traders to make sure that they have the exposures to inflation and market volatility sought by the bank. Which trader has taken an appropriate position among the traders you are monitoring? @, Trader A bought a call and a put, both with 90-days to expiration and with strike price equal to the existing spot level. b, Trader 8 bought a put option with a down-and:in knock in feature. Trader C bought a call option at the existing spot levels and sold a call at a higher strike price, both with 90-days to expiration, d, Trader D sold a call and bought a put at the existing levels, both with 90-days to expiration. Copyright © 2009 Global Association of Risk Professionals 86 All rights reserved. ©OGARP LO0N a30CUTON OF RS OnESIONAS 2009 FRM Practice Exams CORRECT: C As the strategy popularly known as the bull spread will result in positive payoff when the spot rises. As inflation increases, spot levels in commodities are expected to rise. Selling a call at higher leve] will reduce the cost of the strategy. Although it may limit the upside, but that would be in line with the view as only a moderate rise is expected in spot. INCORRECT: A~As the strategy popularly known as a straddle is to be used when the view is that the volatility in the market will rise, and there is no directional view on the spot. INCORRECT: 8 ~ As the above option will be suitable when the spot is expected to fall from the existing levels. INCORRECT: D — As the payoff in this case is similar to short position in spot and would make sense when the underlying is expected to fall Reference: Hull, Chapter 10. 35, Considering options generally (., not only plain vanilla calls and puts), which of the following. statements about vega is correct? ‘An option holder can never be vega negative ‘A deep in the money up and out call option has a negative vega. A deep out of money up and out call option has a negative vega. A deep out of money digital option has a negative vega. CORRECT: 8 Deep in the money Up and Out call option because an increase in the volatility of such options leads to the increasing chances of option either being knocked out (ifthe price increases beyond the barrier) or loosing its moneyness (ifthe prices falls) and hence the increasing volatility tends to have negative impact on the price of the option. INCORRECT: A ~As an option holder can be Vega negative as shown above. INCORRECT: C— have positive Vega as an increase in the volatility would increase the chances of getting towards moneyness and hence positive Vega from a holder’s perspective. INCORRECT: D — have positive Vega as an increase in the volatility would increase the chances of getting towards moneyness and hence positive Vega from a halder’s perspective, Reference: Hull, Chapters 17,18 24. Copyright © 2009 Global Association of Risk Professionals 87, All rights reserved. 2009 FRM Practice Exams 36, To hedge against future, unanticipated, and significant increases in borrowing rates, which of the following alternatives offers the greatest flexibility for the borrower? Fixed for floating swap Interest rate collar Interest rate floor Call swaption aoge CORRECT: D The question focuses on flexible management of borrowing expenses. While a fixed for floating swap could reduce borrowing expenses, it's @ fong-term contractual commitment to exchange payments. if interest rates decline, the borrower may gross up to the agreed fixed rate. An interest rate collar is @ combination of an interest rate floor and cap, ie, it locks in the interest expenses within a tight range, Moreover, collars usually offer interest rate protection at one particular point of time unless several contracts with different maturities are exchanged, A call swoption gives the company the right to enter into a swap when the borrowing expenses exceed a certain reference rate. If the reference rate is below the borrowing expenses, the option is not exercised. Reference: Hull. 37, Assuming other things constant, bonds of equal maturity will still have different DVO1 per USD 100 face value. Their DVO2 per USD 100 face value will be in the following sequence of highest value to. lowest value: 2. Zero coupon bonds, par bonds, premium bonds b. premium bonds, par bonds, zero coupon bonds © Premium bonds, zero coupon bonds, par bonds d._ Zero coupon bonds, premium bonds, par bonds CORRECT: B DV01 is certain multiple of Dirty Price (which includes Coupons) and not Clean Price. Thus, it is proportional to Base Price, which is Dirty Price. Ordinarily, Premium Bond will have the highest (dirty) price followed by Par Bond and with the least price of Zero Coupon Bond. Hence, DVO1 of Premium Bond is the highest while that of Zero Coupon Bonds is the lowest. INCORRECT: A ~ Premium Bond will have a higher Base Price and hence higher DVO1 than that of Zero Coupon Bond. INCORRECT: C— Base Price of Par Bond is higher than that of Zero Coupon Bond and hence, its DVO1 cannot be less than that of Zero Coupon Bond. Copyright © 2009 Global Association of Risk Professionals 88 All rights reserved. ©OGARP oma sssciunox orm rnresiaats 2009 FRM Practice Exams INCORRECT: D — DVO1 per USD 100 Face Value is an Absolute Amount of USD based on actual Base Price Change. Ordinarily, Base Price of a Zero Coupon Bond will be lower than that of Par & Premium Bond. Hence, DVO4 of Zero Coupon Bond is less than that of Premium Bond of same maturity. Reference: Tuckman, “Fixed Income Securities”, Chapter 5. 38. The information ratio of the Sterole US Fund for 2006 against the S&P 500, its benchmark index, is 1. For the same time period, the fund's Sharpe ratio is 2, the fund has a tracking error of 7% against the S&P 500, and the standard deviation of fund returns is 5%, The risk: free rate in the US is 4%, Calculate the return for the S&P S00 during the time period. a 35% b. 7% ce 11% d. 14% CORRECT: B Sharpe Ratio = 2 (Fund Return —Risk Free Rate)/SD = 2 (Fund Return ~ 4%)/596 Fund Return = 14% Information Ratio =1 (Fund Return ~ S&P 500 Return)/ Tracking Error = 1 (14% - S&P 500 Return) /7% =1 S&P 500 Return = 7% INCORRECT: A ~ Incorrectly divides S&P 500 Return by 2 INCORRECT: C~ The candidate might use the Tracking Error as the Numerator in both the Ratios, Sharpe Ratio =2 (Fund Return ~ Risk Free Rate)/Tracking Error = 2 (Fund Return ~ 4%6)/7% = Fund Return = 18% Information Ratio = 1 (Fund Return ~ S&P 500 Return)/ Tracking Error = 1 (18% - S&P 500 Return) /7% =1 S&P S00 Return = 11% INCORRECT: D ~The candidate can stop with the fund return calculation, and end up with 14%, Sharpe Ratio = 2 Copyright © 2009 Global Association of Risk Professionals 89 ‘llrightsreserved. 2009 FRM Practice Exams (Fund Return ~ Risk Free Rate)/SD = 2 (Fund Return ~ 4%)/5% Fund Return = 14% Reference: Amenc and Le Sourd, Portfolio Theory and Performance Analysis. Chapter 4 39. A fund manager recently received a report on the performance of his portfolio over the last year. According to the report, the portfolio return is 9.3%, with a standard deviation of 13.5%, and a beta of 0.83. The risk-free rate is 3.2%, the semi-standard deviation o\(R,} of the portfolio is 8.4%, and the tracking error of the portfolio to the benchmark index is 2.8%. What is the difference between the value of the fund's Sortino ratio (computed relative to the risk-free rate) and its Sharpe ratio? 0.274 1727 0.653 0.378 aoge connect: o(R,) 13.5% Sharpe ratio equals to R,-R, 5 While Sortino ratio equals to 2 =F. = 9-370 —3.2%4 o(Rp) 8.4% Tracking error is used to calculate the value of the information ratio, which is defined as R, -R, +2. The calculation of information ratio is not required in this question. a(R, —R,) 727 .653 (0.073 = (.093 ~ 0.032)/0.83) 1.378 INCORRECT: B= 2.178-0,452 INCORRECT: C~ 0,726-0.0.73 = INCORRECT: D~ 0.073~0.45: Reference: Amenc and Le Sourd, Portfolio Theory and Performance Analysis. Chapter 4 Copyright © 2009 Global Association of Risk Professionals 90 All ights reserved. OGARP SOHAL sstcaron oF msn PovessonaLs 2009 FRM Practice Exams 40. Your firm has no prior derivatives trades with its counterparty Super Bank. Your boss wants you to evaluate some trades she is considering. In particular, she wants to know which of the following, ‘trades will increase your firm’s credit risk exposure to Super Bank: 1 buying a put option il. selling a put option Ill, buying a forward contract IV. selling forward contract a. |.andilonly b. Mand IV only c. Mland IV only d._ |, Illand Iv only CORRECT: D This tests understanding of the type of positions that create credit risk (and links to the following question). The key is to evaluate each of the component trades. Buying a put option creates credit risk. Buying or selling forward contracts creates credit risk, However, selling an option does not create credit risk you are not subject to the performance of the counterparty. Reference: Hull Chapter 22, Stulz Chapter 18. 41. Consider the following one-period transition matrix: Initial Next Period State Period A 8 Default State a 95% | 5% 0% B 10%] 80% | 10% Default 0% 0% 100% fa company is originally in State A, what is the probability that the company will have defaulted strictly before the fourth transition period from now? 0.875% 0.500% 1.375% 1.875% pees CORRECT: C Copyright © 2009 Global Association of Risk Professionals 92, All rights reserved. @OGARP SLO ABORTION OF ik HOFESIONNS 2009 FRM Practice Exams The easiest way to determine the answer would be to make this a square matrix including default in initial state. Then self-multiplying the matrix three times yields three-period transition matrix. We can also manually do the calculation; After yeor 1 there is a 0% chance of default and 5% chance of being in state B. After year 2 there is 95%*5% + 80%*5% chance of being in state B and 5% * 10% chance of default. After year 3 there isa (95%*5% + 80%*5%)*10% additional chance of default. Answer A assumes just one year INCORRECT: A ~ Only considers the third year transition from B to default. INCORRECT: 8 - Only considers the second year transition from B to default. INCORRECT: D ~ Mistakenly doubles the second yeor transition from B to default, Reference: De Servigny and Renault, Measuring and Managing Credit Risk, Chapter 2, Appendix 2A, page 49 ~ 52. 42. As an approximation, itis true that Default swap spread = Return of a risky bond + Return of a risk-free bond Default swap spread = Return of a risky bond — Return of a risk-free bond Default swap spread = Return of a risky bond x Return of a risk-free bond Default swap spread = Return of a risky bond x (1- Return of a risk-free bond) aooe CORRECT: 8 The buyer of a risky bond can hedge the credit risk of the risky bond using a default swap. Entering into the swap trade reduces credit risk. To preclude arbitrage, the buyer of the risk bond has to receive the same return as the risk-free asset, or: Return of a risky bond = Return of a risk-free bond + Default swap spread Reference: Hull, Chapter 23. 43. Ina CDO, the SPVis typically AAA-rated Acrated BBB-rated Not rated apoe CORRECT: A Copyright © 2009 Global Association of Risk Professionals 92 All rights reserved.

Anda mungkin juga menyukai